Wednesday, January 02, 2019

Electrons don’t think

Brainless particles leaving tracks
in a bubble chamber. [image source]
I recently discovered panpsychism. That’s the idea that all matter – animate or inanimate – is conscious, we just happen to be somewhat more conscious than carrots. Panpsychism is the modern elan vital.

When I say I “discovered” panpsychism, I mean I discovered there’s a bunch of philosophers who produce pamphlets about it. How do these philosophers address the conflict with evidence? Simple: They don’t.

Now, look, I know that physicists have a reputation of being narrow-minded. But the reason we have this reputation is that we tried the crazy shit long ago and just found it doesn’t work. You call it “narrow-minded,” we call it “science.” We have moved on. Can elementary particles be conscious? No, they can’t. It’s in conflict with evidence. Here’s why.

We know 25 elementary particles. These are collected in the standard model of particle physics. The predictions of the standard model agree with experiment to best precision.

The particles in the standard model are classified by their properties, which are collectively called “quantum numbers.” The electron, for example, has an electric charge of -1 and it can have a spin of +1/2 or -1/2. There are a few other quantum numbers with complicated names, such as the weak hypercharge, but really it’s not so important. Point is, there are handful of those quantum numbers and they uniquely identify an elementary particle.

If you calculate how many particles of a certain type are produced in a particle collision, the result depends on how many variants of the produced particle exist. In particular, it depends on the different values the quantum numbers can take. Since the particles have quantum properties, anything that can happen will happen. If a particle exists in many variants, you’ll produce them all – regardless of whether or not you can distinguish them. The result is that you see more of them than the standard model predicts.

Now, if you want a particle to be conscious, your minimum expectation should be that the particle can change. It’s hard to have an inner life with only one thought. But if electrons could have thoughts, we’d long have seen this in particle collisions because it would change the number of particles produced in collisions.

In other words, electrons aren’t conscious, and neither are any other particles. It’s incompatible with data.

As I explain in my book, there are ways to modify the standard model that do not run into conflict with experiment. One of them is to make new particles so massive that so far we have not managed to produce them in particle collisions, but this doesn’t help you here. Another way is to make them interact so weakly that we haven’t been able to detect them. This too doesn’t help here. The third way is to assume that the existing particles are composed of more fundamental constituents, that are, however, so strongly bound together that we have not yet been able to tear them apart.

With the third option it is indeed possible to add internal states to elementary particles. But if your goal is to give consciousness to those particles so that we can inherit it from them, strongly bound composites do not help you. They do not help you exactly because you have hidden this consciousness so that it needs a lot of energy to access. This then means, of course, that you cannot use it at lower energies, like the ones typical for soft and wet thinking apparatuses like human brains.

Summary: If a philosopher starts speaking about elementary particles, run.

728 comments:

  1. Tim,

    You do not understand what I say and therefore wrongly conclude that I contract myself.

    Physics can not tell a being how it feels to be a different being. Neither can anything else. You may call that a problem. I consider it to be unsolvable. The best you can do is find approximations based on the observation that being A is similar to being B.

    What is not a problem for physics is to explain that a large amount of particles does have experiences in the first place. Not that I am claiming someone actually can practically compute it. I am merely saying that there is nothing needed in addition to physics. Hence my reaction to people who claim that the human capacity for love proves there must be more than physics and such.

    ReplyDelete
  2. Sabine,

    Unsolvable problems are, ipso facto, problems. And if it is unsolvable, then physics cannot solve it.

    Physics cannot explain why "a large amount of particles does have experiences". If it could, then in principle you could calculate—from physical principles—how many particles it takes and what state they have to be in to have an experience. And you cannot, even in an idealized, non-practical way, provide even a vague clue about how such a calculation would go. That is because there is a hard problem of consciousness that is beyond the scope of physics per se.

    If you disagree, then give us the cash value: what pure physics calculation, even in principle, will reveal how many particles it takes to create a conscious being? (Maybe the answer is 1, and the panpsychists are right.)

    ReplyDelete
  3. So a particle IS its properties. Anything that has the properties of an electron is an electron, and it makes no sense to talk about "what it really is", right?

    ReplyDelete
  4. Tim,

    "Unsolvable problems are, ipso facto, problems. And if it is unsolvable, then physics cannot solve it."

    That's correct.

    "Physics cannot explain why "a large amount of particles does have experiences". If it could, then in principle you could calculate—from physical principles—how many particles it takes and what state they have to be in to have an experience. And you cannot, even in an idealized, non-practical way, provide even a vague clue about how such a calculation would go. That is because there is a hard problem of consciousness that is beyond the scope of physics per se."

    You are making big claims for which you have no backup. Sure you can in principle calculate what it takes for a collection of particles to have an experience. The number of particles is not the relevant factor though. I don't know what you think prevents such a calculation other than practical difficulties and lack of good data about what goes on in brains.

    ReplyDelete
  5. Please provide some vague clue about exactly what it is you are calculating. I want to know if the lobster feels pain or not. What do I calculate?

    ReplyDelete
  6. Tim,

    How can I make this any clearer? An answerable question is "Does being A have an experience?" a non-answerable question is "Does being A have the same experience as I (or you)?"

    "I want to know if the lobster feels pain or not. What do I calculate?"

    I don't understand the question. If you want to know whether a lobster has an experience of any kind, to begin with you'll have to make an observation, preferably of that lobster's brain. A calculation alone will of course not suffice.

    If you have an observation then you compare the result of that measurement with your calculation.

    You may be asking what kind of calculation you compare your measurement with, but of course no one knows for sure. That's exactly the kind of problem I think would be interesting to answer. I offered some guesses above but I don't seriously expect anyone to agree with those. Really my point is merely that it's an answerable question.

    Eg, you might be looking for an activated pattern in the lobster's brain that predicts a situation it will try to avoid. Though for a lobster it's probably mostly hardwired, so it may be simpler than that. In any case, I hope it's clear that I'm not a neurobiologist and don't know a terrible lot about lobsters. I just don't see what's there not computable about it.

    ReplyDelete
  7. Sabine,

    You can make it clearer by actually giving any details at all, rather than throwing around empty verbiage. I am hereby granting you complete access—down to the individual quark and election—to the physical state of the lobster's nervous system.

    Hypothesis 1: that physical state and its activity are accompanied by a felt sensation of pain.

    Hypothesis 2: that physical state and its activity are purely unconscious and autonomous, with no accompanying sensation at all.

    OK: I am handing you the physical state on a platter. In all physical detail. What do I calculate to decide between the two hypotheses?

    ReplyDelete
  8. Tim,

    That's a brilliant question, we should try to answer it.

    Look, I can merely tell you how I believe one could go about answering the question. I do not seriously expect my guess to actually be correct or be useful in the future as it is admittedly a topic I know very little about. My point is just that it's an answerable question and we should try to answer it because a lot can be learned from it.

    I was guessing that the difference between a conscious and an unconscious state is that the former is monitored while the latter is not. For this reason the latter state also will not make use of circuitry that's necessary for predictive modeling.

    Now, neurobiologists have made a lot of progress in recent years in identifying what parts of the brain play a role in conscious awareness, and really I don't follow this literature, so I cannot tell you whether what I say makes a lot of sense. Again, my point is merely that it's possible to do. You can very well encode the prescription I just suggested as example into a computer and figure out whether a brain activity has or doesn't have this property.

    Though I doubt you are asking this question because you actually want an answer.

    ReplyDelete
  9. TomH

    Don't do this to me! You said:

    "I'll note in passing, that Philip Pullman's fantasy novel trilogy "His Dark Materials", posits a particle called "dust" -- later in the trilogy "dust" is also called "dark matter"."

    First, note that I'm not a "mathemagician" or whatever they call those people who like to hang around oil refoundries for fun. But I have long wondered (and I pose this question to anyone who might quickly chime in before The Queen bans me forever [I know... BUSTED]):

    Can the apparently-missing "dark matter" be explained by "dust" (in actuality, any small traces of matter) at concentrations say of one unit per cubic kilometer of "empty" space? Would such concentrations result in any apparent opacity at astronomical distances? Most importantly, could such concentrations account for the missing mass?

    In effect, is interplanetary/stellar/galactic space REALLY empty, or are those (almost) innumerable units of "emptiness" able to hide that much mass in plain sight?

    If this notion can be dismissed out-of-hand, please, someone disabuse me of it so that I might go on with my life...

    R. Taylor

    ReplyDelete
  10. Sabine,

    As soon as one tries to get down to actually physical principles that would account for consciousness, this is the sort of problem that arises. Does consciousness have something to do with "monitoring"? OK, that it not impossible. But the very concept of "monitoring" is not a term of physics. One could argue that any pair of entangled particles "monitor" each other because there are correlations between their behaviors. But if that is correct, then it seems at least possible that a single pair of entangled electrons could be conscious in some sense because each is "monitoring" the other. And that is awfully close to panpsychism.

    Of course, I am not expecting an actual answer from you! The hard problem is hard—the hardest problem there is—and we haven't even touched the question of how one could acquire evidence that monitoring of any sort really is the key to producing consciousness.

    My point is that you expressed puzzlement about why anyone would think there is in principle (rather than merely in practice) a deep problem in accounting for the evident fact of consciousness from purely physical principles. I have been trying to articulate why it is such an intractable problem, whose intractability lies much deeper than merely the difficulties in carrying out calculations that we know in principle should hold the answer to these questions. We not only don't know what to calculate about a physical system, we don't know how to go about determining whether a proposal for what to calculate is the right proposal, and those problems arise from the conceptual gap between the third-person and the first-person.

    But I agree in that I don't see panpsychism as making any real progress here. The problem with the hard problem is that the best argument for any position is an argument by elimination: show that every other proposed solution has intractable problems. But everyone can argue for their position that way, since every single approach appears to have intractable problems.

    ReplyDelete
  11. Can we consider evolution as consciousness? Trying and trying until managing...

    ReplyDelete
  12. Tim,

    What I mean by monitoring is that the system has self-similarity. It can "predict itself" to some approximation. Since the link seems to have gotten lost, I wrote about this in some more detail here. If nothing else, you may find it amusing.

    In any case, I am not puzzled that anyone would think it's difficult. If it was easy, we'd have done it already. I am puzzled at people who declare it must be impossible because they prefer to think there must be something about consciousness that the laws of physics cannot account for. And I am afraid that this belief stands in the way of making progress that could help many people, hence my issue with panpsychism.

    That there are some questions that will remain unanswerable (what does it feel like to be a bat, does a lobster feel pain like Tim Maudlin does, etc) is for me really not so interesting. More interesting is that there are questions which I think are answerable, if not now then in the soon future.

    ReplyDelete
  13. Sabine, which comment are you responding to here? If the one you didn't publish, it was indeed a caricature so let me apologize and thank you for not publishing it. But I think it would be beneficial if you had a less cavalier attitude towards philosophy, an academic field which is not the one you're expert in.

    Otherwise I've mainly tried to be constructive about this discussion and may continue of you allow me to.

    ReplyDelete
  14. Sabine,

    "It is clear that (a) you did not understand what I wrote and (b) you nevertheless believe you know more about QFT than I do."

    So you assumed in (a) that what I'm saying didn't show an understanding but offer no details so basically you're asking to just believe you and shut up. It's like if I respond that clearly you didn't understand what I wrote in my comment... would that be a productive answer?

    Then you assumed (b) that for some reason it's not important what I said but it's who I am and whether you think I believe I know more QFT than you do? Maybe I do, maybe I don't, why does it matter since the little I said is pretty universal in QFT?

    Of my comment though you ignored the actual attempts that exist where people in facts do look into this. Is that a dismiss? Some people clearly are trying to actually do actual quantitative science on this, not philosophy.

    Not sure why my comment, came with good intentions, became an opportunity to be treated poorly simply because I don't agree with you. Basically I was told: "shhh, Ruggero, you don't get it obviously, but even though you believe you're so smart." Very welcoming.

    Why personal comments rather than commenting on the topic? If you don't care to answer you can ignore me, no need to be snarky and personal about it.

    Cheers

    ReplyDelete
  15. Bee and Tim, if you're interested in viable solutions that have been published in peer-reviewed journals, of which there are a number extant now, check out, e.g.: http://integratedinformationtheory.org/. IIT provides a quantitative framework for calculating the amount of consciousness in any collection of objects. It also provides a toolkit for characterizing individual experiences as "constellations" of points in a type of qualia space. So there are extant frameworks that do exactly what you both are saying can't be done. You may not agree with this (these) solutions, but they are out there.

    Our similar approach provides a simpler quantitative framework for calculating the amount of consciousness present in any putative combined consciousness (but we haven't developed a qualitative evaluation tool yet). This is our general resonance theory of consciousness (GRT). IIT and GRT are both panpsychist in their assumptions because in IIT it's integrated information that is the seat of consciousness and all matter has at least some iota of integrated information (phi), and in GRT resonating fields are the seat of consciousness and all matter is simply a specific kind of resonating field.

    Penrose and Hameroff's Orchestrated Objective Reduction theory (Orch OR) posits that microtubules and similar proteins are the seat of consciousness because they allow for quantum effects to filter up to the macro level of the brain. Orch OR is panprotopsychist in its assumptions.

    In terms of testing panpsychism as the basis for a theory of consciousness and comparing it to purely materialist physicalist models, there are various ways to flesh out such a framework. I have an in progress paper on this and the central method of the proposed testing framework is to begin with the most obvious cases of conscious experience — people like you and I — and to gradually move down the chain of complexity, and by doing so to gather evidence that will support or refute the panpsychist framework proposed. So rather than proof, we should and can seek support or refutation/falsification. And we shouldn't forget that the only consciousness we know for sure is our own, all else is based on reasonable inference.

    ReplyDelete
  16. Quentin,

    If you want to have a constructive discussion, how about you not make statements about my supposed deficiencies that are demonstrably incorrect. I just literally wrote a whole book about metaphysical principles that are being used in physics, and about the question in which cases it's a good idea to use them.

    I also would suggest you comment on what I actually wrote rather than complaining about what I did not write. This is a blogpost which makes one single point and not a review article about panpsychism.

    ReplyDelete
  17. Steven Evans,

    Your comment here was very thoughtful and sound, much appreciated. But I am afraid this comment here could start another personal fight between the Stevens, that made Sabine close the comment section here. Please both Stevens resist the temptation. Might the hard wired “fine-tuning ==> Creator” one stay the thoughtful Dr. Jekyll. Thanks in advance.

    ReplyDelete
  18. Ruggero,

    Your remark that my argument would hold in a classical world demonstrates you do not know how quantum numbers enter the calculation of observables.

    Your comment that Feynman diagrams are about "experience" further demonstrates that you do not understand those calculations are normally done using those very diagrams.

    I have no desire to teach you QFT, there are many good textbooks on it, hence the brevity of my response.

    ReplyDelete
  19. bee:

    i just want to thank you for allowing discussions in the comment section to go 'off-track' unlike other scientific bloggers who censor their readers and unnecessarily limit their comments. personally, i enjoy hearing and reading anything 'smart' people have to say about anything even if they're wrong (i.e. don't agree with me)Feynman was right when he said

    "I believe that a scientist looking at nonscientific problems is just as dumb as the next guy"

    but i'll take smart people saying dumb things over dumb people saying dumb things (just as i prefer science bloggers who have contributed to the field they criticize over science bloggers who only contribution is their criticism. those who have 'walked the walk' at least have 'street cred' when' talking the talk').

    naive theorist

    ReplyDelete
  20. Sabine Hossenfelder said...

    "Consciousness is an act of information processing. I don't understand why some people think there's something mysterious about it. "

    We probably won't agree about this, though clearly we do agree that panpsychism is the wrong answer, as I mentioned earlier.

    Suppose you go to the dentist and agree to have a tooth drilled without a local anaesthetic. Compare that with what happens if you do have an anaesthetic - you process the same information, but your consciousness perceives the situation very differently.

    Put another way, your computer processes huge amounts of information, but do you consider it is conscious?

    I followed your link, and I do indeed see a kindle page for your book, but a panel on the right hand side says "This title is not currently available for purchase". Since others are experiencing the same problem, I guess it isn't me, but you may be losing a lot of losing sales because of this glitch - can't you, as the author, remonstrate with Amazon?

    ReplyDelete
  21. One good thing about panpsychism is that it acknowledges one major problem with "emergence theory": it doesn't follow from anything we know about physical laws that they should lead to the emergence of subjective experience. That applies to currently known physical laws, and also arguably to as yet undiscovered physical laws, because (as some commenters have already pointed out) physical laws by their very nature can only describe the way things interact with each other.
    It may still be the case that subjective experience "emerges" from the properties of matter, but then this emergence must follow from some additional laws that come on top of physical laws.
    In the case of panpsychism, the additional laws state that all "things" have subjective experiences.

    ReplyDelete
  22. I’m appreciating this conversation even though it seems like it might be a bit painful. But surely it’s important.
    If we choose to keep both the physical world and consciousness as being real, then it seems to me that physics will be incomplete until it can integrate conscious experience into the material world. Consciousness appears to be unobservable to the current tools of physics, except in correlation with particular dynamics within the nervous system. It seems reasonable to expect that we will eventually map a correlation between specific molecular structures and their dynamics to specific qualia and motor activation. Considering that consciousness is a holistic unity of many signals plus an adaptive world model, we will need to have an integrative model in addition to the qualia mapping. The holistic part could conceivably be via the pattern in the connectome, but it could also be related to collapse of a quantum superposition state. Of course the brain is supposed to be too warm and wet for that, but we are finding other biological systems that utilize quantum effects, so I’m not sure it can be ruled out.

    ReplyDelete
  23. Tim,

    Maybe to get a better grip on consciousness and weather it is or isn´t a hard question just a short question.
    Consider a neural network (NN), i.e. software running on a classical computer. Will a complex enough NN ever be able to evolve consciousness? This means do you think consciousness is a (weakly) emergent property?

    ReplyDelete
  24. Sabine wrote:
    "You are a lot of particles and that's that. A lot of particles that deny they are a lot of particles."
    So, apparently, electrons cannot think, but they can deny...

    ReplyDelete

  25. Team Φ: "Consciousness is information processing."
    Team Ψ: "Particles have psychical properties."

    May the best team win. :)

    ReplyDelete
  26. Tam Hunt

    Apologies, I was sloppy in how I expressed myself. But I still don't think you are addressing the issues of:

    Why would you refer to the resonance of atoms and molecules as a form of consciousness, when there is no evidence that it is anything but physical resonance?

    The faster information flows in a brain that you are claiming "lead to more complex consciousness" are produced by the physics of the matter in the brain. There is no need to posit that this matter has any properties beyond what physics says, and Sabine Hossenfelder says it can't display any additional behaviour according to the empirically confirmed physics.

    It seems to me that when all the superfluous and unjustified meta is removed, you are simply saying that physical matter in the brain produces fast information flows via resonance which leads to consciousness. And even this I think is just a vague connection, because if you knew in any detail how matter was producing consciousness, you could produce consciousness artificially. Which I presume you can't.

    So I think your ideas boil down to: fast information flows via resonances may be an important factor in producing the consciousness we see in a brain.

    Maybe.

    ReplyDelete
  27. Reimond said...

    "But I am afraid this comment here could start another personal fight between the Stevens"

    I have asked Steven Mason and Philip Goff how "panpsychic" matter produces consciousness in the brain. Steven Mason claimed that panpsychism explains consciousness but didn't detail how, and Philip Goff pointed to a link on an intro to monism at the end of which it is admitted that the theory cannot explain how "panpsychic" matter produces consciousness in the brain, which was the goal of the theory....

    If people are claiming that panpsychism explains consciousness in the brain, they should be able to give us an outline of how, because it looks like a failed theory.

    I'm all ears...

    ReplyDelete
  28. David,

    "Suppose you go to the dentist and agree to have a tooth drilled without a local anaesthetic. Compare that with what happens if you do have an anaesthetic - you process the same information, but your consciousness perceives the situation very differently."

    A local anesthetic blocks pain receptors, usually by flooding sodium channels. If you do that, your brain does not receive the same information - that's the whole point. What do you think how local anesthetics work, magic?

    Look, I have nothing to do with the amazon pages of my book. Besides that, for me it's all looking fine. Yes, there are several people who have problems, they tend to be located in the UK. The problem with being in the UK is being in the UK. I do not have a publisher in the UK. Best,

    B.

    ReplyDelete
  29. I am rather struck by how many people seem to be in favor of panpsychism, which is something that strikes me as being in a way sort of new aged woo-woo. This is making the waves and I noticed today that Lubos Motl wrote a pro-panpsychism piece on on his blog with a rather appalling concluding remark.

    Brent Meeker wrote the following in an email on this topic, which I think has a bearing on this:

    I noticed that no one even considered the problem of how particles with
    qualia can be aggregated to produce human consciousness and then have
    that consciousness vanish with a little stoppage of blood flow or
    anethesia[sic]. That makes it plain to me that consciousness is
    operationally coestensive [sic] with intelligence, i.e. evolutionarily useful
    behavior in response to an environment. I think B makes a good point is
    saying it is relational. Without the relation to the environment,
    intelligence can't be defined. We know a lot about intelligence and AI
    research is adding to that knowledge. So I think we base a measure of
    consciousness on what there is an intelligent relation to. To complain
    that this doesn't tell us what consciousness IS is just a confusion.
    Science doesn't tell us what anything IS, it just tells us how it
    behaves, how it relates.

    I received news yesterday evening that a friend died, and she was fairly young. One of my brothers committed suicide back in 2012. My most wonderful big bull terrier dog was struck by a truck over a year ago. If you have had these experiences with death you may have been impressed by how a dead body is empty; there is no person there, or in the case of a dog there is no longer a dog-being in there. To carry on with Brent's comment quoted above, a change in physical processes can result in the irreversible end of consciousness.

    There are lots of arguments being made here for and against panpsychism. I have considered or contemplated this idea in the past, as I presume most of us have. A lot of things have been brought to the table, from hidden variables in quantum physics to Gödel's incompleteness theorems. Brant Watson wrote an extensive piece and mentions Gödel's incompleteness theorems. There is then an interplay of concepts here with quantum information (qubits), complexity, and self-reference. I might be able to write a serious book on this, and at the risk of either being glib or conversely weighty here goes.

    (continued exceeded limit)

    ReplyDelete
  30. CONT: First off let us dispense with hidden variables. The Bell-Kochen-Specker theorems illustrate how if hidden variables exist they must be nonlocal. In other words there can't exist local hidden variables with classical (like) properties that account for quantum mechanics. John Bell showed quantum physics violated classical inequalities and the Kochen-Specker theorem illustrates that if local hidden variables exist then 8 = 9. For details, look this stuff up.

    A quantum measurement has properties similar to self-reference. A measurement system is composed of a large number of quantum states, and it is used to measure a prepared quantum system usually with a small number of quantum states or degrees of freedom. The quantum phase of superposition or entanglement of the observed system enters into the quantum states of the measurement system. So we could say a quantum measurement is a sort of self-referential loop. The contradictions between various quantum interpretations is also remarkably similar to aspects of ω-inconsistency if one imposes completeness.

    Gödel's incompleteness theorems have been appealed to with respect to consciousness. One aspect of consciousness is the idea of there being a self-directed awareness of the “I,” or that “I exist.” There is however a possible category error here if one assumes this is the same as the quantum case. In general Gödel's incompleteness theorems are for classical Boolean systems, though they can be used to look a linear space of states of QM. The leap from consciousness to quantum physics is a big category equivalency that is not obviously justified, and likely wrong.

    One observation test of quantum consciousness is that if mind where quantum then the eyes as extensions of the brain would also be. This means our vision should have quantum telescopic properties as an interferometer. The average distance between eyes is about 7cm, which means if we looked up at the night sky we should be able to see that planets as if looking through a 7 cm telescope. We don't have this ability.

    I now shift of complexity. Entropy, action or the Lagrangian and complexity are all related to each other. Quantum information may be the most fundamental quantity conserved in this universe. This carries into black holes and conserved quantum information or BPS charges on horizons. So the argument would then be information behind our consciousness has been existent all along. That is true. The quantum information behind the construction of the brain and the processes that generate consciousness have been around all along, and continue after death. The quantum information contained in my i-phone has been around all along, and if while visiting Hawaii I should drop it into a lave flow that quantum information still exists. However, the i-phone is gone. Qubits has massive combinatorics for entanglements and interactions with other qubits. Scramble things up and large scale objects defined by certain categories of constructions from fundamental information bits or qubits are gone.

    I must conclude and get actual work done. Consciousness is an emergent property, just as a metamorphic rock such as gneiss is emergent.

    ReplyDelete
  31. Opamanfred,

    I already explained above that a set of elements can very well have properties that the single elements of the set do not have. No single particle can think. A lot of them together can. I don't know why that is so hard to comprehend.

    ReplyDelete
  32. Sabine,

    "Your remark that my argument would hold in a classical world demonstrates you do not know how quantum numbers enter the calculation of observables."

    Hilarious. Based on what? What did I say wrong about quantum numbers? These are again personal attacks adding your own interpretation of my words rather than looking at the meaning of what I was saying.

    "Your comment that Feynman diagrams are about "experience" further demonstrates that you do not understand those calculations are normally done using those very diagrams. "

    Even more hilarious, since I've computed a ridiculous number of those. If you had an open mind on my comment, you'd have understood that I am referring to the fact that panpsychism is about consciousness and experiences, and that interactions are basically how they include experiences in fundamental objects (or sometimes not quite that but things along similar lines - there are many of those theories around). I certainly didn't mean that Feynman diagrams in QFT are experiences, I simply meant something obvious, which is that cross sections are really central to the concept of observables, so if someone assumes that an electron has some non zero level of consciousness, its interactions or self-interactions are often considered by those studies its experiences. But you're assuming not sure based on what that my understanding stops at the word quantum numbers.

    Words in comments or posts are often imprecise, so we should tend to assume good faith, not the opposite. For example, if we need to be super picky about quantum numbers then there is possibly even something imprecise with respect of how you chose to count particles in the standard model.

    "I have no desire to teach you QFT, there are many good textbooks on it, hence the brevity of my response."

    I don't like to say this, because I feel it's bad taste on my side since my studies shouldn't matter, but I have a PhD in HEP-ph and my thesis and papers were on SUSY. But you seem to have the desire to be extremely patronizing and not interested at all in what one of your readers was trying to actually tell you. Nice scientific curiosity on your side, if you think you already knew already all you needed to know about a topic where many many very smart people dedicate a lot of time on...

    I was hoping in your second message you'd have seen that you made too many bad assumptions about my first message, but it seems instead that you chose to double down on the prejudice that I must have been ignorant on both panpsychism and QFT, as well as wrong.

    I can only conclude that either you don't welcome real discussions assuming good faith on comments, or maybe you suspended that after responding to too many comments.

    Why are you so angry? Weird...

    Cheers

    ReplyDelete
  33. Ruggero,

    You came here to "educate" me about Feynman diagrams. You continue to try to "educate" me, this time by bragging about your PhD. Interestingly enough, you further complain that I am the one who is "patronizing." Maybe read your own comments.

    If you were trying to make an argument, I can't find it. Sorry, but I don't have time for this nonsense.

    ReplyDelete
  34. Sabine,

    You say "Physics can not tell a being how it feels to be a different being. [...] What is not a problem for physics is to explain that a large amount of particles does have experiences in the first place."

    I'm puzzled by this statement. To me, the problem of knowing what experiences other people or physical systems have is nearly the same as knowing whether they have experience at all. I think both problems are equally intractable. Of course I assume that people who are a bit like me also have experiences, but I also assume their experiences are more or less similar.

    What makes you think that these two problems are different and that one is easier than the other?

    ReplyDelete
  35. Tam,

    I don't think you are quite following what I am pointing out as the center of the Hard Problem.

    Of course someone can propose some calculable physical quantity and then say it measures consciousness. Tononi did. But the next question is whether it is the right quantity to calculate. That is, is it really what creates, or is, consciousness? I could say that the rest mass of an object is a measure of its consciousness, and now you have a calculable number, but on what grounds should we accept that that is even relevant?

    One response is to say that it is just obvious that, say, the Rock of Gibralter is not more conscious than I am despite having a much, much larger rest mass. Personally I am convinced by that. But what if the proponent insists that it is? How can one decisively refute the suggestion?

    Similarly, Tononi produced a calculable number. Aaronson showed that according to that proposal, many laptops doing sorting tasks are more conscious than humans. I am convinced that that refutes Tononi. But what if Tononi just bites the bullet and says that yes, the laptops are more conscious? How does one refute that?

    Panpsychism strikes me as similar. I do not believe that individual electrons are conscious at all, so any theory that implies that they are is wrong. I admit I cannot prove that. But the point is that the panpsychist cannot really offer even a shred of evidence for such an extraordinary proposal. Of course, I cannot *refute* the proposal if the panpsychist is just willing to bite whatever bullets have to be bitten, just as I cannot *refute* the proposal that rest mass measures consciousness if someone is willing to just bite the bullet that the Rock of Gibralter is hyper-conscious, and I cannot *refute* Tononi if he is willing to bite the bullet that my laptop is (not could be, but is) more conscious than I am.

    I have myself offered what I take to be a decisive argument that consciousness cannot be accounted for by computational structure ("Computation and Consciousness"), but someone could again just bite the bullet and insist that moving a block of wood that is not in contact with anything else half an inch in a vacuum can make the difference between feeling a toothache and feeling nothing. I personally find that result so unacceptable and implausible that it is essentially a reductio ad absurdum, but I do admit I have not actually derived an absurdity from the theory: just a result I find unacceptable. And the result that a single election is conscious—even a little bit—I find equally unacceptable. And that the Rock of Gibralter is much more conscious than I am. But these are not absolute proofs, and there can be no absolute proof in this business, no matter how much physics I know and granting that the physics as physics is perfectly correct. That just reflects why the Hard Problem is hard. And why physics qua physics cannot solve it.

    ReplyDelete
  36. Quentin,

    Science works by finding patterns. We have experiences and we can try to understand what pattern of brain activity corresponds to an experience - in fact people already do that. Same with consciousness - people are trying to understand what are signals of a conscious/unconscious thought pattern. I cannot see anything in the way of tightening this connection.

    Now, look, you may one day very well be able to answer the question whether person X has an experience of (say) falling. Whether that person's experience feels the same as yours, on the other hand, is a question that I think is unanswerable for the reason I laid out above - just posing the question is a contradiction. You cannot be someone else. Though I am guessing that if your brain circuitry is similar enough to the other's, you may be able to get a pretty good impression of some things. If we are lucky to live long enough, we may even be able to try out 2nd hand experiences.

    In any case, this is why I think it's helpful to distinguish the two cases. Thanks for the question (and also for the earlier apology).

    ReplyDelete
  37. Reimond,

    If you ask me whether a neural net running on silicon can become conscious, the answer is I haven't a clue! I do accept that consciousness supervenes on the physical structure of an object, so I reject philosophical zombies as metaphysically possible. And I know from first-hand subjective experience that my own brain has whatever it takes to support consciousness. And I also have an argument (again, in "Computation and Consciousness") that whatever that feature is, it is not computational or information-processing structure as articulated in terms of Turing machine structure. But if you require that a Neural Net be a particular kind of physical instantiation of a computational structure (so not just a Turing machine characterization), then I just have no clue. Maybe that's the key to consciousness. But then again, as Searle suggested, maybe the key to consciousness (like the key to lactation) lies in organic chemistry, and silicon just doesn't have what it takes, no matter how it is arranged.

    The problem is that although I know that my brain supports conscious states, I don't know what generic feature of my brain is responsible for that. So I can't know whether a Neural Net running on silicon can instantiate that feature.

    The Hard Problem is Hard!

    ReplyDelete
  38. @Tim Maudlin: Agreeing with your rejection; I have to reject panpsychism on the same basis I reject religion; it is apparently intentionally designed to make unprovable claims and then expound upon the consequences of them as if they were undoubtedly true. Like claiming we are "too complex" to have evolved without help, or the world or solar system or universe is too complex. (All provably false claims of course).

    Panpsychism seems exactly the same, based on unprovable claims, intentionally built into those claims are unobservable states and properties. It hardly matters what people can "prove" if their proof rests on unprovable claims and unobservable phenomenon that they nonetheless proceed to describe, in order to prove something they cannot define or measure (consciousness).

    The absurdity is their notion proving too much [See Wikipedia], I can use their same tools to prove that the first matter in the universe took the form of a Unicorn, for a Planck unit. You will just have to accept a few other unobservable things I can't prove or measure.

    ReplyDelete
  39. Lawrence Crowell,

    "First off let us dispense with hidden variables. The Bell-Kochen-Specker theorems illustrate how if hidden variables exist they must be nonlocal. In other words there can't exist local hidden variables with classical (like) properties that account for quantum mechanics. John Bell showed quantum physics violated classical inequalities and the Kochen-Specker theorem illustrates that if local hidden variables exist then 8 = 9. For details, look this stuff up."

    This is a misunderstanding of Bell. Bohmian mechanics is a "hidden"-variables theory that has no problem with Bell's theorem, and one that Bell himself strongly appreciated and advocated. But the "hidden" variables—namely the particle positions—are local, not non-local (and they are manifest, not "hidden"). What is non-local in the *ontology* of the theory is the quantum state, but, as the PBR theorem proves, you can't get around taking the quantum state ontologically seriously.

    What Bell proved is that the physical world—actual physics, whatever it really is—is non-local. He did not in the least rule out "hidden variables", and was indeed the strongest proponent in the world for them.

    ReplyDelete
  40. sabine,
    In your sentence:
    "But if electrons could have thoughts...",
    could you please give an example for the "collision" you refer to, and explain how the number of particles changes if electrons could think?
    thanks

    ReplyDelete
  41. sabine and others,

    what philosophers who espouse panpsychism are attempting to address is how the brain which is made of matter can experience consciousness. some variations of the idea is not that a single electron has consciousness but that as a part of nature it is a part of something that has consciousness.

    re: your objections,

    the double slit experiment using a single electron could be a way nature is showing "consciousness"

    ReplyDelete
  42. Tim,

    With Turing machine you (probably ;-) mean a deterministic one.
    I definitely want to include randomness in the process of the neural net as is state of the art, which in terms of Turing I guess must be more like a probabilistic Turing machine .
    And with the restriction “… running on a classical computer” I just wanted to get away from Hameroff and Penrose Orch OR and also far away from any panpsychism.
    I just do not want the maybe emerging consciousness tied in any way to the hardware. But of course, some hardware is needed for the software to run on. In case of silicon QM is underneath, but does not play any role. Just a classical computer implemented in whatever hardware.

    And yes, I also agree it is a hard problem, but one which has obviously one solution – us.

    ReplyDelete
  43. Rolf,

    I merely mean if electrons had additional degrees of freedom you would have to find a way to encode those in the Lagrangian and sum over them if you calculate the outcome of a scattering event. The point is simply you cannot postulate a particle has additional properties without indeed allowing the particle to make use of them. You are asking for an example of a "collision" but really any will do provided the energy is high enough (see remark about strong coupling). It also doesn't matter, of course, that I was referring to electrons in particular, same thing for any particle.

    If you are familiar with the literature, there's a similar argument for black hole remnants (or rather against them). The issue is that those would have to exist in huge numbers, ie they have many internal degrees of freedom. This means that even if the probability to produce any one of them is small, you would produce them in huge numbers because there are so many of them. It goes under the name "pair production problem".

    Now, look, there are ways to try to circumvent this conclusion - as I keep saying, any proof is only as good as its assumptions. My issue is merely that if you want to give new properties to elementary particles you change the standard model and it's not all that trivial to make that compatible with observation.

    ReplyDelete
  44. Classical Philosophers were doing Literature ... But safely, We can call Contemporary Philosophers: Staticians ... You just have to add lot of "measurements/data to justify your literature.

    ReplyDelete
  45. @neo: ...how the brain which is made of matter can experience consciousness

    That implicitly assumes that "consciousness" cannot be produced or experienced by any form of matter. Where is the proof of that? Just because it "feels different" doesn't prove anything. As far as we can tell experimentally, all consciousness we observe is intimately tied into matter, and specifically networks of neurons. If we disrupt that network (by damage, starving it of fuel, etc) then consciousness is lost.

    Where is the proof that "consciousness" has nothing to do with matter or operating neurons?

    As a computer scientist, this is akin to a novice asking me "But where are the ones and zeros that are being manipulated"?

    There are no actual ones and zeros independent of the underlying hardware, they are encoded by low and high voltages, circulating charges in memory circuits, and the transformations are composed of clever XOR devices (transistors) that can take a high voltage as an input and produce a low voltage as an output, or vice versa.

    The same idea applies here to thoughts and feelings; they do not exist independent of the underlying neurons and synapses, thoughts are neurons changing state, thoughts are the electrochemical signals occurring on nerves and axons and synapses, as are feelings and pain and "qualia" and "love". They are not separate from the matter, they are matter.

    ReplyDelete
  46. I don't think it is an accident that the consciousness we can be sure of that it exists, namely our own human consciousness, is associated with an incredibly complex state of matter, called a brain. In comparison, elementary particles, at least as far as we know it, are just darn simple.

    ReplyDelete
  47. @Sabine @Tim Maudlin
    I have followed your interesting discussion. I do side with Tim when he says that no physics can provide a full explanation of subjective experience.
    Perhaps this example can clarify. Take a written word or sentence. You can analyze the chemical composition of the ink and the paper, the physics of how the ink disperses in the structure of the paper, etc, to finer and finer details (down to elementary particles). None of that will ever tell you anything about the meaning of the word or sentence. For that, you need to switch to a different level of description, in terms of grammar and syntax instead of particles and fields.
    In the same way, a different level of description is needed to describe consciousness - in terms of meanings, intentions, will, decisions, and so on, which is not the language of physics.

    ReplyDelete
  48. Opamanfred,

    Those are emergent properties. The can be derived from physics, at least in principle. Again, I do not understand why people think there is something supposedly impossible about this. Not only is there no reason to think it's impossible, such impossibility would necessarily be in conflict with the theories we know to be correct already.

    ReplyDelete
  49. Sabine,
    Did you read what I wrote? You mean the meaning of a word "emerges" from the chemical composition of the ink in which it's written? I do not understand.

    ReplyDelete
  50. In a comment above by Lawrence Crowell:

    "Science doesn't tell us what anything IS, it just tells us how it behaves, how it relates."

    Doesn't this leave science deficient (in its study of consciousness)?

    ReplyDelete
  51. @ Tim Mauldin Bohm's mechanics is somewhat applicable to nonrelativistic QM. Since it “clings” to a classical framework it is maybe useful for quantum chaos. The particle or beable has features that appear nonlocal and there is a Hamilton-Jacobi equation for the wave ψ = ρe^{iS} from the Schrodinger equation that has this additional potential -ħ^2Δρ/ρ. This is where locality has difficulties, and so far Bohmians tend to adjust this quantum potential nonlocally to measurements. Bohm's interpretation is only effective if this quantum potential is regarded as nonlocal. So far I have not seen how this is made into a local hidden variable that completely reproduces QM. I know there is a fellow, Sheldon Golden I think, who is laboring tremendously on this. However, as yet I see no cigar. The problem with nonlocal hidden variables is they can be disposed of as excess baggage. They really contribute nothing.

    The real difficulties come with relativistc QM. Take the Klein-Gordon equation □ψ – m^2ψ = 0 with □ = ∂^a∂_a. Let me write the polar wave as ψ = e^{r + iS} with ln(ρ) = r. The KG equation has real part

    ∂_aS∂^aS - ∂_ar∂^ar - □r - m^2= 0,

    where the last two parts form the quantum potential and the imaginary part is

    □S + 2∂_aS∂^ar = 0,

    that is the pilot wave. We have that ∂_aS∂^aS = ∂_tS∂^tS - ∂_iS∂^iS or E^2 - p^2. So we have the momentum-energy interval

    m^2 = E^2 - p^2 - ∂_ar∂^ar - □r.

    For the mass zero we can see that something funny happens where the quantum potential is acting like a mass on the light cone. This also can lead to the motion of a beable outside the light cone. This is a reason that Bohm's QM is not regarded as consistent with relativity.

    Other practical issues stem from the fact it is not based on Hilbert space of states and it has been difficult to get it to reproduce the generation of particles, such as e-p pairs in QED. These are old problems well solved since Feynman and to a good extent before him.

    So Bohmian mechanics is a very limited interpretation. I think it may be useful for quantum chaos problems with the quantization of chaotic classical systems. It is not hard to derive the path integral from it in a Feynman-Hibbs sense. However, I think Bohm does not really overturn the cart of nonlocality by wiring it up with local beables and the rest. Interpreting it as a nonlocal theory is alright, but nonlocal hidden variables are just excess baggage to be rid of.

    ReplyDelete
  52. Sabine.. I have to ask.. Way above you say something like "I don't believe in qualia". What can you mean by that? Your entire moment by moment conscious arena comes out to a quale. You cannot be saying that your experience as such isn't real?

    ReplyDelete
  53. You can in a sense say that the particles in my/our brains have consciousness, because they are part of a system that has consciousness.

    I understand from the theory of quantum mechanics that you can switch two random identical particles around and the wave function remains the same, except that for fermions the sign changes.

    Therefore all the electrons (or other particles) in my/our brains are the same as every other electron in the universe. Therefore all electrons are conscious through me/us.

    ReplyDelete
  54. Dr. Castaldo,

    It appears as if you are making Neo's point:

    To paraphrase Neo:

    "How can the brain, which is made of matter, experience consciousness if the components of that consciousness are not contained within the matter?"

    I don't see this position as implying that, since the brain is made of matter, it can't possibly experience consciousness. Quite the opposite.

    You reply, again, paraphrased that:

    " ... thoughts and feelings ...are not separate from matter, they are matter."

    Aren't those two positions in agreement, outside the fact that Neo may be referring to more basic constituents while you are referring to slightly more complex ones?

    R. Taylor

    ReplyDelete
  55. Sabine,

    You say "We have experiences and we can try to understand what pattern of brain activity corresponds to an experience".

    But we can as well try to understand which pattern corresponds to which kind of experience. Now it might be just a label that we know how to interpret (to link to memories), such as "free falling" or "fear" because we already had such experiences. But I would say that similarly, "experience" is a generic label that we know how to interpret because we do have experiences. So I don't really see the difference.

    ReplyDelete
  56. I came here with a positive attitude, good faith, and an interesting (to me) reference and how it pertains to the problem.

    You had a negative attitude, bad faith on my intentions, and were very rude to me. You can also look back at your comments and see that.

    This is your blog, you could have ignored my messages instead of being negative and rude. No one deserves to come here to talk about something that interests them in a respectful way and be treated so poorly.

    ReplyDelete
  57. Hey folks. I’ve been following this discussion with great interest (and commented once somewhere in there). Anyway, I’m trying to come up with a [less brief than I thought] summary for myself, and I thought it might be useful to post here, so here goes:

    1. Sabine has stated that panpsychism, the idea that everything, including electrons, have some consciousness, adds nothing to our understanding of the world.

    2. Different people have different ideas as to what differentiates “consciousness”. Some people say an attention mechanism differentiates consciousness. Some people, possibly including Sabine, require higher order thought (thoughts about experiences). Some people require “awareness” of the environment, although these people usually also require a consequent appropriate response. For example, chemotaxis (moving toward or away from a stimulant) in a bacterium would satisfy. These people could be called Functionalists, as the response to the environment serves some function. Some people consider “awareness” to be the ability to respond in any way to something in the environment. This last group would be panpsychists. (I’m not saying all panpsychists fall into this category. I’m saying all who fall into this category are panpsychists.)

    3. We know matter by what it does. For any given chunk of matter, we describe it by what it does (how it effects light, how it pushes on other things, etc.). More specifically, we know it by what we see/feel/measure it does. In some cases we have to insert intermediary chunks of matter, like bubble chambers, to see it do what it does.

    For any given chunk of matter, we can sometimes see sub-chunks, often by smashing them really hard into other chunks. But again, we only know these sub-chunks by what they do. At some point we get down to chunks which we can no longer break into sub chunks. We can try to postulate some mechanism that explains why these chunks do what they do, but the only way to verify would be to predict that under some circumstance (smashing with just lots more energy?) we would see something new. But that would just leave us with smaller chunks. You will always have a bottom level, a level which we have not observed any sub-parts. At this level, it makes no sense to try to ascribe a mechanism, because there would be no way to differentiate that mechanism from angels.

    4. Panpsychism seems to be giving the name “consciousness” to the mechanism that makes particles do what they do. That’s fine, but that would be no different than giving the name “angel” to the mechanism that makes particles do what they do. The particles will still do what they do. If this is correct, Sabine (#1) seems to be correct.

    Tam Hunt seems to be proposing a specific mechanism (synchronizing vibrations) to explain how particles do what they do, which is cool, and hopefully testable, but that still leaves us with particles doing what they do.

    5. Combining particles (doing what they do) in specific organizations will produce emergent properties. One of these properties is functionalist awareness, which can be observed in bacteria and in neurons. Further combination of such functionally aware stuff can produce further emergent properties, such as qualia and higher order thought.

    It’s possible Tam Hunt’s hypothesis will explain how the low level stuff is combining into emergent properties we didn’t know were happening (vibrational synchronization across the brain?), but then he will have to explain how that leads to qualia, higher order thought, attention, etc.

    6. If we can explain qualia, higher order thought, etc., using what we already know about particles and what they do, panpsychism doesn’t really help.

    *
    [don’t know if that helps you, but it helps me]
    [James of Seattle]

    ReplyDelete
  58. @Opamanfred: The problem with your analogy is that panpsychists are not "switching to a different level of description"; they are claiming the meaning of the word is in the ink and the paper and the physics of ink dispersal and paper construction.

    They have not switched at all, they are claiming the electron is "conscious" in some sense, and so are all other fundamental particles.

    Your example disproves your thesis!

    What is true is they should move to a different level of description; very far away from the particles and into the realm of molecules (neurotransmitters) and cells (neurons) with about $10^{14}$ atoms in each.

    ReplyDelete
  59. A biological neuron processes information in quite complicated ways.
    Inputs to the neurons body are processed in two ways at least.
    One way generates a voltage pulse which travels to the neuron's soma
    or cell body and if there is enough of a voltage change to push the voltage
    at a special place on the cell body called the axon hillock far enough above
    the normal equilibrium voltage maintained between the outside and inside
    potential of the cell, special gates for sodium and potassium open and close
    in just the right way to generate what is called an action potential.
    This rapidly rises to about 60 to 70 mV above nominal and then decays past
    equilibrium and then slowly rises back up to equilibrium. This wave form
    is then transmitted without loss to the input side of other neurons due to special
    biological machinery in the output line from the neuron which is called the axon.
    After this signal is generated the neuron can't generate another signal for
    a specific time unit called the refractory period.

    The second way inputs are processed is that they determine a signal which
    is transmitted to the nucleus which causes the transcriptions of one or many
    proteins which are then used in the neuron. The first way to process an input
    is called a First Messenger and the second way is the Second Messenger.
    Second Messenger inputs effectively alter the biological hardware of the neuron
    so that it processes incoming voltage waveforms differently. There are many
    other things going on but this explanation is a start. The voltage waveform
    transmitted along the axon of the neuron enters a special structure we can call the synapse near the input cable of another neuron. This causes a calcium +2 current
    spike inside the synapse which if large enough causes packets of neurotransmitter
    inside the synapse to migrate to the cell walls of the synapse, bind to the wall
    and popout into the gap between the synapse and the input cable of the input neuron.
    This release is quantal as it is in integer number of packets.

    So output neuron -> voltage wave -> synapse ca +2 current -> quantal neurotrans
    release. The packets then bind to a gate on the input cable and generate
    a first or second messenger reponse. Hence, there is a lot of signal transduction here and you should not think of this as analog/ digital.

    ReplyDelete
  60. A small comment on consciousness. It is known 50K or so surgical patients
    become iZombies where they retain memories of the surgery at various levels
    even though anesthesized. The anesthesia's purpose is to change the level
    of consciousness of the patient in such a way that recognition of ongoing
    sensory input is greatly reduced. This is not understood very well so
    so discussions of consciousness, albeit strangely theoretical, do have
    practical consequences for all of us. Also, clues about the nature of
    consciousness come from the drastic changes induced in the prey (cockroach)
    of a predatory wasp. Upon injection of a neurotoxin cocktail, the normal
    behaviour patterns of the cockroach are altered and it allows itself
    to become a larder for the wasp's young. This is also hard to understand
    but is useful information for the construction of cognitive models as a cognitive
    model should be able to show the same alteration in behavior.

    Clearly it is very hard to build these models but some are trying.

    ReplyDelete
  61. Sabine,

    I think the current philosophical discussions on the problems of consciousness are due to a recognition that "explaining" consciousness by terms such as "emergent" or "information" does not work. I.e., it is just about as hard to explain "emergence" or "information" in purely physics terms as it is to explain consciousness. (I actually have a good deal of professional knowledge of "information theory" -- e.g., some patents in the field: it's a very interesting and very useful part of mathematics and computer science, but it is hard to connect it to physics -- though many, going back to Brillouin, have tried.)

    Have you read anything by Galen Strawson such as this, where he makes a distinction between "physicSalism" and "physicalism." It seems to me a useful distinction, even if you disagree with his final conclusions, as you and I both do.

    I have been following Tim Maudlin's work for some time: among philosophers, he is one of the good guys who takes physics and physicists seriously and wants to try to communicate with us, even if differences in terminology and methods can make that difficult. So... if you are willing to seriously engage with any philosopher at all, he is a good one to talk with.

    Finally, I think your argument in your original post is strong and that it is worth pounding this into the heads of the brighter philosophers. I myself certainly do not understand the nature of consciousness but pointing out real problems with current ideas is actually progress. Philosophers above all people should be aware of this. It will be interesting to see if we can get any philosophers to see the force of your argument.

    Dave Miller in Sacramento

    ReplyDelete
  62. sabine,
    we're not at "circumvent the conclusion", yet. You have not even explained how you conclude:
    "we'd long have seen (a violation of predictions of the SM)." from "an electron can think". Clearly a quantitative argument is needed. What is it?
    thanks

    ReplyDelete
  63. And having had their ideas utterly refuted, Tam Hunt and Philip Goff wander away to carry on publishing nonsense. Unbelievable.

    ReplyDelete
  64. Dr. A.M. Castaldo asked:
    >Where is the proof that "consciousness" has nothing to do with matter or operating neurons?

    That is a bit of a “straw man”: pretty much everyone agrees that consciousness has something to do with neurons! The question is exactly how this works, whether physics as we now know it can fully explain what is happening, and what sort of substrates in general can serve as a basis for consciousness. Those are real, and so far unanswered, questions. (For the record, I do not think panpsychism is the right answer!)

    ReplyDelete
  65. Opamanfred,

    The meaning of a word does of course not emerge from the ink and the paper, it emerges from the context in which it is being used and from how people think about it. We associate words with certain patterns in our brain which again are associated with certain situations and things and so on. Same thing with meaning and intention and decisions, etc. It's all emergent - and it's emergent from physics. You don't need no extra magical fairy dust. Best,

    B.

    ReplyDelete
  66. Quentin,

    Not sure I understand the question. Yes, you can of course identify patterns and match them with reported experiences. Again, that's something people already do. If you get really good at it, you may actually be able to predict what experience someone will report who displays a certain pattern of activity. That may work because human brains are structured similarly. Will you (as in you-the-person) ever be able to check that indeed someone else's experience is the same as yours? No, you won't.

    It's pretty much the same problem that prevents you from ruling out solipsism. You can certainly demonstrate that the assumption of an external reality is very useful and that predictions based on it work well and so on. But will you ever be able to actually prove that anything exists besides you thinking it exists? No, you won't.

    In any case, I just mean to say that while there are questions that will remain unanswered, this doesn't mean that consciousness is all that mysterious.

    ReplyDelete
  67. Ruggero,

    You came here to talk down to me, now you are offended I don't take shit. If you want me to ignore your comments, I suggest you don't post them.

    ReplyDelete
  68. Rolf,

    I already answered this question. You have to account for the additional degrees of freedom in the theory and sum over them if you calculate the outcome of an experiment. I will not repeat this once again and I will not approve another pointless repetition of a question I already answered.

    ReplyDelete
  69. Matthew,

    What I am saying is that I have never come across a definition of "qualia" that remotely made any sense to me. Let us take the Wikipedia entry. It says it's an "individual instances of subjective, conscious experience" like "what it is like to taste a specific apple, this particular apple now".

    As I said above, the only reasonable way to answer this question is to document that that's what this person's brain is doing when eating an apple and that's the kind of experience typically correlated with that pattern, etc. Let us stick with the example of tasting an apple. I am pretty confident that that is actually an experience that you could simulate by poking someone's brain in the right place (people have reported weirder things in open brain surgery) - not that I think you could do it in practice right now. In any case: What is it like to taste an apple? Poke-poke. That's what it's like. Question answered.

    Yet, people tend to use the word "qualia" to emphasize that that state of the brain is supposedly not all there is to say about it. At which point I just have no idea what they could possibly mean.

    As I said above, you will of course never know whether you taste an apple the same way as I taste an apple, but that does not seem to be the issue here.

    Well, I hope this explains why I have difficulties making sense of this. Best,

    B.

    ReplyDelete
  70. @Sabine: "What is it like to taste an apple? Poke-poke. That's what it's like. Question answered."

    I think that's probably true with the caveat that it might be slightly different pokery for each individual. The wiring is slightly different in each brain, and the wiring is everything (or so you would assert).

    FWIW, perhaps this has been said already, the thing about qualia is that we have them at all. It's not that a juicy apple causes some part of our brain to activate; it's that we find the experience meaningful... or that we find it anything at all.

    ReplyDelete
  71. Chris,

    Ah, see, here we go again. Finding an experience "meaningful" is an experience. I don't see what else you think there could possibly be to it. It's a brain pattern.

    And, yes, I already said several times that since no two brains are alike you will never know if someone else's experience is indeed the same as yours. I also said that since human brains do have many similarities there are quite plausibly many experiences that are transferable to some extent. You can approach this with the scientific method in the usual way. I don't see anything in the way of doing that.

    ReplyDelete
  72. @Sabine: "Ah, see, here we go again."

    I apologize. There have been too many posts here for me to keep up.

    "I don't see what else you think there could possibly be to [qualia]. It's a brain pattern."

    Understood. The point is that (as the phrase of art goes) "there is something it is like" to experience qualia. As opposed to there being nothing it is like. My computer (as far as I know) does not experience the data processing it does. There is nothing it is like to be my computer.

    There is something it is like to be me. Or you. Or any of us.

    I agree there is no way to know what the experience of others is like (other than by their word of mouth, which suggests it's not completely different).

    I also completely agree with the scientific method, and of course it can be applied to what it is like to experience qualia!

    ReplyDelete
  73. Chris,

    " "there is something it is like" to experience qualia" "

    I have no idea what that is supposed to mean. I am sincerely sorry, but as I said above already, I have never been able to figure out what people might possibly mean by "qualia". We have experiences, okay, I get that. You seem to be saying now that qualia is to experience an experience, which doesn't make sense.

    ReplyDelete
  74. Unknown wrote: I DID give an example of how heroic materialism can actually undercut science

    Okay, let me see if I understand: Absolute materialism leads to superdeterminism which precludes randomness which precludes evolution which undercuts science.

    Unknown wrote: Superdeterminism thus becomes nearly identical with Fate or intelligent design

    If you roll dice, the result appears to be random to humans even though deterministic forces are acting on them. No need to invoke a supernatural entity or an intelligent designer.

    ReplyDelete
  75. Qualia is the experience. Any experience.

    What makes them interesting is that we have them. It's not clear that we need them, and we have no idea why we have them at all.

    As evolved biological machines, why are you mindful of what an apple tastes like? It's an evolved response to an apple being good for you nutritionally, survival-wise, but why should the experience be pleasant? Why should it be anything at all?

    All our machines do their function fine without experiencing anything. It could be the same with us. Some people wonder why it isn't.

    ReplyDelete
  76. Chris,

    Well, if qualia is the same as experience then I don't see what's the problem with accounting for it in terms of brain patterns. Also, why not just call it experience then?

    ReplyDelete
  77. PhysicistDave said...

    "Have you read anything by Galen Strawson such as this,"

    BS red alert - it's more gibberish from yet another crank. Oh no, he's British - an embarrassment to the nation and wasting my taxes.

    TL;DR of linked gibberish: We most accept the reality of conscious experience, this is the most fundamental fact of nature, therefore physics can't explain consciousness.

    Unless it turns out that it can, of course. Neuroscientists have not exhausted their analysis of the brain. And the only information we have about the brain and any links between subjective conscious experience and the brain has come from science.

    Philip Goff, Tam Hunt and now Galen Strawson tell us nothing. Thousands and thousands of pages telling us nowt.

    Panpsychism, monism or any other consciousness theories that depend crucially at some point on properties of matter other than those demonstrated by physics have been refuted by SH's argument, unless they can show the empirical results of physics are wrong.

    PhysicistDave, you need to change your user name to NotVeryGoodPhysicistDave.

    ReplyDelete
  78. Chris,

    Also, how do you know that machines do not experience anything? They probably do, just not very much. Eating is pleasant because finding it pleasant makes us seek it out, which a survival advantage. I am sure you know that. Again I don't see what's mysterious here.

    ReplyDelete
  79. Steven Evans,

    Tone it down. There'll be no second warning.

    ReplyDelete
  80. @Sabine: "Also, why not just call it experience then?"

    Qualia is the term of art. "Experience" is a general term, including much. Qualia refers to more basic building blocks. The canonical example is what it is like to experience the color red.

    @Sabine: " I don't see what's the problem with accounting for it in terms of brain patterns."

    No problem at all.

    The behavior of a computer is fully explained in terms of its electrical patterns. And, as you've said, human behavior is fully accounted for by our brain patterns.

    @Sabine: "Also, how do you know that machines do not experience anything?"

    In the panpsychism sense? Or in the sense I mentioned in my first comment, like an electron "experiences" a magnetic field?

    @Sabine: "Eating is pleasant because finding it pleasant makes us seek it out, which a survival advantage. I am sure you know that. Again I don't see what's mysterious here."

    Well, yes, I said so.

    The disconnect seems to be in what a computer (or electron) can "experience" and whether by "experience" we mean the rich "something it is like" experience of a juicy apple.

    If I follow, you see it all on a spectrum, that the computer has some "experience" of reality that is, functionally, no different than your experience of eating an apple? Our rich sense of experiencing a juicy apple is just due to our complexity?

    If so, I think we differ in our view of the emergent mind. I believe there is something to our experience (and also, for instance, my dog's experience) that far transcends anything a computer is capable of. (Douglas Hofstadter has an interesting take on it in his I Am A Strange Loop book.)

    ReplyDelete
  81. Hello Dr H.

    Thank you for your reply - I'm glad you take time to reply to everyone's comments.

    I think it isn't really the motive of these philosophers to write about non-empirical things. Generally the problem is set forth like a dilemma (Chalmers characterizes it as "the Hard Problem"): how do we get minds and experiences out of a purely physical world? Many philosophers viscerally react that simply to say the mind and expressions are physical is like giving up, that it's like saying that there is no such thing as minds and experiences, just brains and chemicals.

    I think this is silly, and the amount of back bending performed in order to obfuscate things enough to make your dualist or panpsychist theory not sound so dualistic or panpsychic is completely ridiculous to me. But it really is, I believe, an effort by these philosophers to "save" the mental from physicalism, and to avoid the actual hard problem of acknowledging physicalism and the existence of minds and experiences consistently. And because they are trying so hard to ensure a non-physical place for mental entities, they end up being like all those angels on the head of that pin.

    ReplyDelete
  82. Sabine-

    Let me ask it this way: Can you conceive of a world just like ours, except that there is no conscious experience? In this reality, our minds would evaluate an apple and deliver a rating with no emotional content. We would seek tasty food only because it had a higher rating, not because it tasted good. We would avoid pain on the basis of knowledge, not hurt.

    There would be no "sense" of taste, touch, hearing, smell. Only information.

    If such a world is a coherent idea, if in particular experience isn't necessary, the question some ask is why it exists.

    ReplyDelete
  83. Chris,

    You use terms like "what it is like to have an experience" which you use in addition to "experience" to begin with. Again that's a distinction which doesn't make sense to me.

    What I mean by "experience" is some yet-to-be-identified brain pattern that corresponds to it. I've mentioned it a few times, but in case the link got lost, I wrote up my thoughts on this here, but really I don't myself take it all that seriously. As I keep saying, I'm not a neurobiologist and know very little about the human brain.

    In any case, sooner or later we'll identify the typical properties of brain patters that we call "experiences". My question is how do you know that machines don't have it?

    No, this has nothing to do with panpsychism of the type that I wrote about, as with distributing an undefined "consciousness" on anything. I can't see the point of doing that.

    ReplyDelete
  84. Chris,

    "Can you conceive of a world just like ours, except that there is no conscious experience? In this reality, our minds would evaluate an apple and deliver a rating with no emotional content. We would seek tasty food only because it had a higher rating, not because it tasted good."

    Again this doesn't make sense to me. The "emotional content" and "taste" is a rating.

    "We would avoid pain on the basis of knowledge, not hurt."

    What's the difference between "pain" and "hurt"? Pain is a reaction that tells you to avoid something. Again it's a brain pattern that's there for good evolutionary reasons. I don't know what else you are looking for.

    "There would be no "sense" of taste, touch, hearing, smell. Only information."

    We have certain sensory input. That alone does not make an experience - we know that much. We process this information in many ways and connect it to memory and so on, all of which combined makes what we call "experience." Can you process information without an experience? Probably. But turns out that experiences have evolutionary advantages.

    ReplyDelete
  85. Reimond wrote: But I am afraid this comment here could start another personal fight between the Stevens

    If you've got something to say about my comment, please say it to me instead of inciting Evans.

    My comment was directed to artuncut. I had no intention of responding to Evans, because his comment made it clear that he completely misunderstood what I wrote and I have no interest in correcting him.

    Let's clear up a couple of other points. In the Martin Rees thread, Evans was having a "personal fight" with three people, not just me. He was insulting three people, not just me. (Me, Phillip, and Sid) Moreover, Evans was insulting Rees and Barnes, and anyone who has religious beliefs.

    Sabine shut down the thread because of massive insults. Only one person was using massive insults. Look at that thread and draw your own conclusions.

    So, let's try again. What if anything would *you* like to say about any of my comments? And this time "resist the temptation" to involve Evans.

    ReplyDelete
  86. @Sabine: "The 'emotional content' and 'taste' is a rating."

    Indeed. As you go on to say, these are deeply associative, complex, and textured, for us.

    Perhaps the question can be framed as asking, given that evolution trends towards low-energy solutions, why is our consciousness so rich, given it could be less rich and we could be equally successful.

    @Sabine: "My question is how do you know that machines don't have it?"

    Machines have first level experience. So do we.

    But we can also think about having that experience. And we can think about thinking about having that experience. And so on as deep as you please, with each level having its own content.

    As you say, experience links to memories, all with their own emotional content and meta-levels. The human neural network is massively connected and very holistic. Neurons process analog data, and there are trillions of them. If nothing else, our brains are many orders of magnitude greater in complexity than any machine.

    I think the difference is transcendent.

    (FWIW, I wrote about this in much more detail then I'd go into here. If you're interested, I invite you to Information Processing.)

    ReplyDelete

  87. "The basic problem is this: emergence seems, at first glance, to be a reasonable enough idea, but when pressed for details it comes up sorely lacking."

    Internet Encyclopedia of Philosophy: Panpsychism: 4. Panpsychism vs. Emergentism
    https://www.iep.utm.edu/panpsych/#H4

    ReplyDelete
  88. Sabine Hossenfelder said...

    "Tone it down."
    Apologies. I shall count to 10 to avoid excitable outbursts in the future.

    ReplyDelete
  89. Sabone wrote:
    >Yes, you can of course identify patterns and match them with reported experiences. Again, that's something people already do. If you get really good at it, you may actually be able to predict what experience someone will report who displays a certain pattern of activity. That may work because human brains are structured similarly. Will you (as in you-the-person) ever be able to check that indeed someone else's experience is the same as yours? No, you won't.

    I think you have hit upon what bothers the philosophers. If I understand you correctly, from your viewpoint if we get a dictionary that tells us what brain states correspond to which experiences, we have basically solved the problem. From the philosophers' viewpoint, we have not solved the problem: we have not explained why this brain state produces that experience or, indeed, any experience at all (many activities in the brain seem unrelated to conscious experiences).

    I think we can understand the philosophers' perspective if we think about the problem of (possible) machine consciousness. If the connection between brain states and experiences is merely a lengthy list without a deeper explanation, then it is hard to see how we could transfer that knowledge to judging machine consciousness. If, on the other hand, we had some deeper explanation -- say, that information processing of a certain sort always leads to consciousness -- then we could apply that deeper explanation to the issue of machine consciousness. (I'm skeptical of an "information-processing" explanation for various reasons: I give it merely as an example of an explanation that might satisfy the philosophers.)

    It does seem that even understanding why people disagree on this topic, often quite bitterly, is a big problem!

    Dave

    ReplyDelete
  90. Steven Mason said...
    "Panpsychism solves the problem of consciousness just like God solves the problem of consciousness (and a lot of other things). It's a solution, but not a useful one."

    ..,7, 8, 9, 10.

    Mr Mason, I see how invoking "God" solves any problem as it is equivalent to invoking magic, and agree with you wholeheartedly to the point where if you were physically present here I would hug you. However, the panpsychists are assuming properties of matter that they want to invoke in an explanation of consciousness, but which strong empirical evidence shows that they cannot invoke. Also, no-one has actually given any details of how matter made of consciousness can produce consciousness in the brain, even if it's allowed to completely ignore the observed laws of nature.

    Does this not demonstrate that panpsychism is not a solution, unless about half of physics is wrong?

    ReplyDelete
  91. Chris,

    "Perhaps the question can be framed as asking, given that evolution trends towards low-energy solutions, why is our consciousness so rich, given it could be less rich and we could be equally successful."

    One answer may be that you are wrong in thinking we could be equally successful without it. Or maybe you are wrong in thinking that we are successful ;) Also, as it's been said many times, natural selection doesn't lead to the evolution of absolutely "best" solution, it merely benefits survival of the best that's come up so far. So, maybe, some experience-less AIs will kick us off the planet tomorrow and that too would answer your question.

    In any case, as I argued in the piece I linked to above, I think that experiences are a side-effect of our ability to model and predict our environment (and, to some extent, ourselves). It seems to me there is a clear evolutionary advantage to this.

    Alas, producing predictive models is not something that computers presently do all that much, so I am tempted to say they probably have very little if any experiences, even though they might be processing loads of information. For the same reason I think that if we manage to improve those codes we'll end up with computers that do have experiences. You're the information-processing person, you'll know that smart modeling can dramatically reduce the necessary computational resources, so again I think there's an evolutionary advantage to this.

    Be that as it may, it is arguably rather wild speculation.

    ReplyDelete
  92. Steven Mason said...

    "I had no intention of responding to Evans, because his comment made it clear that he completely misunderstood what I wrote and I have no interest in correcting him. "

    ..,7, 8, 9, 10.

    Mr Mason, Could you explain how I completely misunderstood what you wrote? I am always eager to learn.

    ReplyDelete
  93. @Sabine: "In any case, as I argued in the piece I linked to above, I think that experiences are a side-effect of our ability to model and predict our environment (and, to some extent, ourselves)."

    Heh, I've actually had similar thoughts.

    As you say, so much of this is up in the air. In Penrose's slightly notorious The Emperor's New Mind he frames the book in a fiction about a thinking computer that I always found cute. How does a computer "feel?"

    If physicalism is correct, this is strictly an engineering problem. Someday we will enact Penrose's scenario, we will turn the switch on a system we believe "thinks" like us (or better, or different, but goodly). And then, as you say, we'll know.

    My belief is that such a machine necessarily must physically duplicate the network of the brain. I do not believe any computer simulation will work (for reasons I covered in the link I gave, and explored in depth in over a dozen posts).

    Very briefly, I believe the mind that emerges from the brain is analogous to laser light emerging from physical substances that lase under the correct conditions. We can simulate lasers numerically, but no photons are created. Likewise, no simulation of the brain can give rise to consciousness.

    It'll have to be Isaac Asimov's "positronic" brains. Which Data in Star Trek also had in homage. Those were physical, network devices.

    BTW: The link you gave me seems to be to a PDF by Sofia Magnúsdóttir? I think, therefore I think you think I am I downloaded the PDF and will read it.

    ReplyDelete
  94. Physicist Dave,

    " If I understand you correctly, from your viewpoint if we get a dictionary that tells us what brain states correspond to which experiences, we have basically solved the problem. From the philosophers' viewpoint, we have not solved the problem: we have not explained why this brain state produces that experience or, indeed, any experience at all (many activities in the brain seem unrelated to conscious experiences)."

    No, that's not what I am saying. A dictionary would of course be a start, but the point is that you want to identify patterns in it. Eg, we do have an experience of having a body, and the map of that body is hardwired to some extent, and it's connected to the actual parts of the body and so on. If you were good enough at reading the wiring, you could figure out what corresponds to the experience of (say) raising and arm, or being weightless or what have you. Same thing for all kinds of sensory input. We decode it, we simplify it, we map it to memory. In principle you can read this out of the wiring. (I don't think the current measurement accuracy is remotely sufficient to do even in principle that and it won't be sufficient for quite some while.)

    (I'm wary of why-questions though. There'll always be "why's" we can't answer. Why anything to begin with, etc.)

    ReplyDelete
  95. Chris,

    Thanks, I will check out your blogpost. Yes, I wrote that piece under a pseudonym, but trust me when I say it's of my doing.

    ReplyDelete
  96. @Sabine: "Yes, I wrote that piece under a pseudonym, but trust me when I say it's of my doing."

    Ha! Something about the sound of the name did remind me of you! :)

    Out of curiosity, what's your take on Mary's Room? (If the piece answers that, never mind.)

    ReplyDelete
  97. Tim wrote: I want to know if the lobster feels pain or not.

    In humans, pain is an injury-avoidance mechanism. Lobsters try to avoid injury, so they must experience *something*. We don't know if animals experience pain the same as humans, and yet we make laws that prohibit the torture of animals. We give animals the benefit of the doubt.

    That being said, people put lobsters in pots and steam them to death, so we don't give them the benefit of the doubt. Everyone can hear them desperately trying to get out of the pot. That's why I don't eat lobsters.

    I've seen kids torture animals like snails and spiders. It's impossible to say how these kinds of animals experience injury.

    Interestingly, there is a sci-fi novel by Ted Chiang, The Lifecycle of Software Objects, in which software entities can experience pleasure and pain. There's one disturbing part in which sadistic humans acquire some of these entities so they can torture them. But it's not a dystopian novel. I recommend the book.

    Tim wrote: what pure physics calculation, even in principle, will reveal how many particles it takes to create a conscious being?

    Short answer: We haven't figured everything out yet. But you already know that.

    ReplyDelete
  98. Re summary:

    I do hope that Particle Physics and Doctors of Philosophy still go together like a horse and carriage.

    Best wishes.

    ReplyDelete
  99. Steven Evans wrote:
    "The "panpsychic" bit is looking a tad superfluous, and the "hard conscsiousness problem", if you think there is one, remains."

    Well the "panpsychic bit" looks a non-starter to me because the Pauli exclusion principle applies to electrons (and protons) so if electrons were conscious, they would all have to be conscious in the same way - which makes no sense.

    The Hard Problem was formulated by the philosopher David Chalmers in 1995, and really lives up to its name. If someone doesn't think the concept is real, I think it is vital that they explain exactly why - preferably without waiving their hands.

    I think qualia are involved in a great deal of thought. I mean part of solving a physics problem is to want to do so - if you don't care, you probably won't get the answer. However, 'wanting' is yet another example of a qualia, and you have to ask yourself how you make a computer 'want' something. If someone says "My computer wants to do X", they obviously mean it in a humorous way - literally they mean that the algorithm in question has a tendency to end up doing X, which isn't the same thing as wanting.

    Those of you who program, try to imagine writing a program that actually wants to do something - whatever you like.

    Hint - it isn't enough to use a PRINT statement to output something like "I want to solve this problem!"

    ReplyDelete
  100. Chris wrote: Can you conceive of a world just like ours, except that there is no conscious experience? In this reality, our minds would evaluate an apple and deliver a rating with no emotional content.

    Does there necessarily have to be emotional content in a conscious experience? Do worms have emotional content?

    ReplyDelete
  101. Sabine wrote to me:
    > A dictionary would of course be a start, but the point is that you want to identify patterns in it. Eg, we do have an experience of having a body, and the map of that body is hardwired to some extent, and it's connected to the actual parts of the body and so on. If you were good enough at reading the wiring, you could figure out what corresponds to the experience of (say) raising and arm, or being weightless or what have you. Same thing for all kinds of sensory input. We decode it, we simplify it, we map it to memory. In principle you can read this out of the wiring. (I don't think the current measurement accuracy is remotely sufficient to do even in principle that and it won't be sufficient for quite some while.)

    Well, of course everyone on all sides of the issue agrees that such research is worthwhile!

    But, suppose we were stupendously successful with that research program. Presumably such research could help us understand the issue of consciousness in other primates, in other vertebrates such as cetaceans, maybe even in invertebrates such as octopi.

    But, would it give us any information at all as to whether machines built out of silicon are conscious?

    I think that is what the philosophers want -- a truly magnificent theory that answers such questions far beyond our current understanding. And, perhaps your point is simply that they are just not going to get that any time in the foreseeable future.

    One way of understanding what some of the philosophers, such as Colin McGinn, are saying is that they are trying to make that point of the extreme difficulty of reaching such a grand theory of consciousness in opposition to some of their older colleagues who thought they had such a theory. E.g., a few years ago, Dan Dennett wrote a book Consciousness Explained: needless to say, he didn't.

    Part of the difference here may be that we physicists do not run into other physicists who grandiloquently proclaim that they have a grand theory that encompasses all conceivable forms of consciousness -- space aliens, machines, black holes, you name it. But philosophers have run into this from their fellow philosophers.

    I'm not sure your views of what can and cannot be understood in the foreseeable future vis a vis consciousness are that different from McGinn's or Maudlin's.

    By the way, thanks for allowing this discussion on a subject that invariably gets people riled up!

    Dave

    ReplyDelete
  102. Sabine,
    You took my example literally, and therefore misunderstood it. It wan an analogy. Here it goes again: if you analyze a word purely from the physicochemical perspective (composition of the ink and paper) you get nowhere in understand it. To do that, you need to switch to a different level of description (linguistic, social, psychological...).
    What I meant is, many philosophers (like TM here) believe the same occurs for consciousness: understanding its physical basis (brain patterns...) is fine, but would not tell us anything about the subjective experience of those brain states.

    A separate problem for the view that consciousness is an emerging property, is that of infinite regression. When does consciousness start to appear? For a worm, an amoeba, a laptop computer? You yourself did not exclude that machines can have a rudimentary consciousness. Then why not a chain of 10 atoms? And, in the end, why not a single electron? I say this to suggest that your own position (emergence) is not too far from panpsychism.

    ReplyDelete
  103. Evans wrote: I see how invoking "God" solves any problem as it is equivalent to invoking magic, and agree with you wholeheartedly

    Yes.

    Evans wrote: Could you explain how I completely misunderstood what you wrote?

    If you understand that God doesn't explain consciousness, then you're halfway to understanding what you misunderstood. Look at my statement and pay particular attention to how I relate panpsychism to God.

    ReplyDelete
  104. Steven Mason,

    My comment was not meant to be a complain about you, not at all – sorry, my fault - I should not have referred to you. On the contrary, I appreciate your comments as I already said here also referring to your comment here.
    Therefore, I thought you would known how it was meant.

    ReplyDelete
  105. @R. Taylor 2:29 said: outside the fact that Neo may be referring to more basic constituents [electrons] while you are referring to slightly more complex ones [neurons]?

    No. Neo said ...how the brain which is made of matter can experience consciousness. some variations of the idea is not that a single electron has consciousness but that as a part of nature it [the electron] is a part of something that has consciousness.

    The general panpsychist claim is that "consciousness" is a separate property of matter, like a new fundamental particle or force, presumably because (IMO) they refuse to accept their feelings, imagination, wonder, love, ecstasy and grief can be accounted for by matter alone.

    This is what Neo is referring to; that electrons contain this funky field that contributes to this magical consciousness we feel. In general panpsychists (IMO) are trying to rehabilitate the notion of a soul in some way, to elevate "consciousness" above matter, beyond the meat of the brain.

    My position is that consciousness (a) lies on a spectrum and (b) is correlated with the complexity (and perhaps accuracy) of internally modeling the external world, including one's self and others, and (c) what we (and I presume many other animals) experience as consciousness are all these millions of models operating in massive parallelism triggering each other, thus creating "lines of thought" to explore problems, simulate solutions, reject them, etc.

    But in the end no special fields or new physics are needed, like the 1's and 0's in a computer are voltages on wires processed by a maze of transistors, our thoughts and minds and feelings are these electric and chemical neural signals processed by a maze of neurons. Our feelings are floods of chemicals (which is why we can hack them with drugs). Our memories are recordings of patterns of activation of our neural models. Our "experience" of seeing red is the activation of thousands of neural models that involve "red" which may include some level of associated (neurally linked) emotional recall. Thus Consciousness is not a separate field; it is just a constantly evolving brain state. Nothing new is needed to permeate all the components of the brain to make it conscious.

    ReplyDelete
  106. @PhysicistDave

    I said (to @neo): Where is the proof that "consciousness" has nothing to do with matter or operating neurons?

    You said; That is a bit of a “straw man”: pretty much everyone agrees that consciousness has something to do with neurons!

    I made an error saying "nothing to do with matter", but not everyone agrees that consciousness has something to do with neurons. The panpsychists (the topic of this thread) propose that consciousness is inherent in electrons and all other fundamental particles, ergo it does not require neurons at all. They assert that all particles have some tiny bit of "consciousness" that can accumulate into some sort of consciousness field through "resonance", perhaps like iron atoms organizing to create a magnetic field or something like that.

    You say: ...and what sort of substrates in general can serve as a basis for consciousness. Those are real, and so far unanswered, questions.

    They are not unanswered! Asking this question presupposes that the substrate we see, neurons and other biological mechanisms in the brain, are an insufficient basis for consciousness.

    This was my point in several posts here; what we see is entirely sufficient and consciousness, thoughts and feelings are just the ever-changing mosaic of signaling going on in our brain as our minuscule neural models are driven (by other neural models) to do something, like think of a better way to factor a matrix, or explain something to a student, or rearrange the garage or whatever.

    In other words I don't believe there is "substrate" problem for panpsychists to solve, other than getting over the fact that we are made of nothing but meat, as if that diminishes our experiences of love, loyalty, beauty, or grief or outrage.

    The real questions are answered, there are just a super-majority of people that don't like the answers or their implications, and thus are constantly inventing new fantasies to try and avoid the real answers.

    ReplyDelete
  107. @James of Seattle,

    Panpsychism isn't about just *giving a name*. I don't know why this idea comes up again and again. We know first hand what it is to have qualitative experience. No observation will ever prove that other people have experience, so attributing experience to other people is somehow idle, empirically speaking: "experience" cannot be a term of a physical theory because it's not defined by its causal role or predictions or anything like this. Yet we *do* understand what the term means and we *do* attribute it to other people and sometimes to animals. The panpsychist attributes it to quite everything, that's what the position amounts to. But that's nothing like renaming properties: it's a substantial metaphysical picture.

    @Sabine

    I agree with what you say. But I fail to see why what you say about "having such or such experience" wouldn't apply to "having experience to start with". To me everything you say apply to both (and your mention of solipsism seems to confirm that): for all I know, other people could all be unconscious zombies that behave exactly like normal people.
    So the problem of knowing which experience someone had is equally intractable as the problem of knowing if they have experience at all. And I still find puzzling (even more so given your reference to solipsism) why you think one problem is easier than the other.

    ReplyDelete
  108. Chris,

    For what I recall (it's been a while) Mary's room is a confusion of terminology as I believe its originator also eventually concluded.

    ReplyDelete
  109. Quentin,

    The reason you can't tell that other people aren't unconscious zombies faking consciousness is that you do not normally know the state of other people's brain. If you did know, you'd be able to tell. (That is, once we'd figured out what marks a conscious state.)

    ReplyDelete
  110. PhysicistDave,

    "But, would it give us any information at all as to whether machines built out of silicon are conscious?"

    Sure it would. I don't see any reason why that consciousness or experience should depend on the material of the network. It comes about from the network connectivity and dynamical laws. Once you know what criteria it takes to make a conscious being, you can figure out whether a machine will fulfill them.

    (Though personally I think the silicon-approach will go nowhere.)

    ReplyDelete
  111. Opamanfred,

    I think you have confused yourself with your analogy. You cannot analyze the meaning of a word from a print of the word on a page simply because that does not contain sufficient information.

    If what you want to say that in practice no one is going to use the standard model of particle physics to analyze the human brain, of course that is correct. That would be totally impractical. We use emergent properties and emergent dynamics and so on.

    "understanding its physical basis (brain patterns...) is fine, but would not tell us anything about the subjective experience of those brain states."

    That's in conflict with reality. Such experiments can be done, and have been done, see this recent example.

    ReplyDelete
  112. First off a bit of an erratum. I wrote to Tim Mauldin The particle or beable has features that appear nonlocal and there is a Hamilton-Jacobi equation for the wave … when the bolded text should read local.

    It is interesting how this topic has caused such a stir. I think this illustrates how the removal of cherished ideas about the centrality of ourselves generates cognitive dissonance. People become very upset by any suggestion their God or Savior Who promises eternal life may not really exist. “What, you mean my infinite gift of eternity is an illusion? How dare you suggest that!” The same this holds for consciousness, and physics has drifted a bit far into these anthropic ideas that strike me as dubious. It is not hard to see a long history of this from the Copernican Revolution to Darwin. The configuration of a stellar system of planets is not unique or fixed, but the result of some initial conditions established in formation. Species of life are not fixed or set in stone, but rather mutable and often erasable. We are the same as is our cherished conscious identities.

    Panpsychism reflects a desire to have our existence scripted into foundations. There is a long history on how these types of ideas end up on the trash heap, even if some still cling to them long after their demise.

    ReplyDelete
  113. David Bailey said...

    ”Well the "panpsychic bit" looks a non-starter to me because the Pauli exclusion principle applies to electrons (and protons) so if electrons were conscious, they would all have to be conscious in the same way - which makes no sense.”

    I don't think the Pauli Exclusion Principle would put the panpsychics off.
    My understanding is that SH's argument against pansychism is that matter cannot exhibit any properties except its physical ones. So whether all matter is assumed to be "experiencing consciousness" or not makes no difference as the only input there can be from matter into the explanation of consciousness is the physical properties of matter or behaviour emergent from these. This leaves the logical possibility that something contributes to consciousness other than matter, though there's no reason to think so, and either way panpsychism and monism are failed theories.

    "The Hard Problem was formulated by the philosopher David Chalmers in 1995, and really lives up to its name. If someone doesn't think the concept is real"

    I don't think anyone is saying it's nailed, just that there's no reason to think it can't be explained as an emergent behaviour of matter.

    ReplyDelete
  114. Consciousness is rather too abstract. Biology starts with self awareness. Such tests are done with mirrors, Most advanced others than human are Orang Utans. These species is estimated to remember, to know about his own personal history. The tests used by science belong most often to the range of disponitional realism. To refresh your philosophical knowledge perhaps lookup wikipedia for realism.

    This whole fills up the path between electrons in dispositional experiments and those exchange between stem cells, neurons and all that brain stuff. There really a whole lot of literature published about panpsychism. There won't be a ether again or an phlogiston. Which both might be if the electron is taken under consideration the very roots of such ideas. I too don't consider entity realism as a brigde or even a hint into the right direction. Fixing the physics beyond SM is rather such a deep panpsychism phenomenon.

    I prefer to stay with those astronoms, that declare, that envidently some really great astronomical entities leaf our consicousness permanently, because their light has now much to far to travel to reach us. There are certain horizons that can not be overcome without the next level an light and electrons are not always the right instruments or experiments to deal with.

    One big topic in which panpychism plays a role is in SM the electroweak particle interaction. It always has to come into consideration if the physics fails, because it can not strongly be desputed. Physics life with context unlike mathematics where the description with context free grammars in regarded highly. Physics is deeply in need for philosophers strong enough to handle the epistemological question of the newest statii of physics. This is some of the biggest demand of philosophical physicists of our time.

    ReplyDelete
  115. @Steven Mason

    "If you understand that God doesn't explain consciousness, then you're halfway to understanding what you misunderstood. Look at my statement and pay particular attention to how I relate panpsychism to God."

    I read what you wrote as the concept of God can explain anything in the sense that the concept of magic can explain anything, but it's really just assumption rather than explanation. Agreed.

    But SH's argument against panpsychism is that even if the panpsychics assume that matter is consciousness instantiated according to the structures of physics, this brings nothing to the party of explaining consciousness in the brain from matter alone, because empirically matter in the brain can only show physical properties. Any assumed panpsychic attributes must remain hidden. So it has been shown that panpsychic assumptions can contribute nothing to an explanation of consciousness, with the same confidence as exists for the results of QM.

    ReplyDelete
  116. Sabine said:

    > (I'm wary of why-questions though. There'll always be "why's" we can't answer.
    > Why anything to begin with, etc.)

    There are two kinds of why-questions. There are the questions that have no good answers and may even be meaningless, such as "why anything to begin with", or (as I think you have argued yourself) why do the physical constants have the value that they have.

    And there are the why-questions which ask why a certain phenomenon follows from known physical laws. For instance, why are there tides? This question can be answered by gravitation.

    It seems to me that this question by physicist Dave should be put in the second category of legitimate why-questions:

    > we have not explained why this brain state produces that experience
    > or, indeed, any >experience at all

    One should figure out whether the fact that certain brain states produce experience follows from physical laws (I don't think so, but I am sure lots of people will disagree).
    And if that fact doesn't follow from physical laws, one should try to find new laws which explain it.

    ReplyDelete
  117. Sabine,

    "Well, if qualia is the same as experience then I don't see what's the problem with accounting for it in terms of brain patterns. Also, why not just call it experience then? "

    "Brain" is not a term of physics. Insofar as it is a term of science at all, it is of biology, and as far as a biologist is concerned, computers simply do not have brains. So if one were to "account for" experience in terms of brain patterns in the sense of proposing that having experiences requires the existence of certain brain patterns, then it would follow that silicon chips cannot have experiences. But you seem to believe that they can. This is an indication that you have not through the implications of what you assert.

    Further, there is not a single "pattern" that exists in a given brain at a given time. Patterns can be described at all levels of detail and in any number of different vocabularies. One "pattern" that is constantly going on in my brain has to do with oxygen being brought in and carbon dioxide being taken out in the bloodstream. Since every investigation in to what it going on in my brain when I have a conscious experience contains that pattern, is that pattern required to have an experience? If so, we get the same result: computers cannot be conscious. Again, contrary to your own assertion.

    So the term "brain pattern"—the thing you appeal to as explaining consciousness—is both hopelessly vague and, taking literally, yields results you yourself reject.

    "Information processing" is no better. As I said, one level at which one can describe how an object "processes information" is the abstract level of instantiating a Turing machine. But to assert that that is what accounts for experience leads to results that are at least as unacceptable as the claim that a single election has experiences. Again: this is laid out in detail in my paper "Computation and Consciousness".

    Then at some point you try a completely different gambit: a very, very naive behaviorism. Pain, you seem to say, occurs whenever a system exhibits avoidance behavior. But that is again at least as unacceptable as the claim that electrons have consciousness. I can design and build a little wheeled robot that avoids the edges of tables. I can understand completely in terms of physics how it works. But if you think that the little robot therefore feels pain, then you believe something as bizarre as that electrons have experience. Precisely because I can account for the avoidance behavior completely by pure physics—without any reference to any felt experience at all—attributing a feeling of pain to the proceedings has zero explanatory power. And if I thought that the little edge-avoiding robot felt pain, I would judge it to be immoral to even build the thing, or at least to let it run on the table. Same question as the lobster.

    One whole aspect of the Hard Problem is that we think that the very felt qualities of experience—the painfulness of pain—play an ineliminable explanatory role in our behavior. We avoid certain things *because they hurt*. But the purely physical analysis is able to account for the physical behavior of avoidance without reference or mention of any felt qualities at all. So the experience appears to be what we call "epiphenomenal": it accompanies the physical activity but plays no explanatory role in producing the behavior. But that seems wrong. I avoid putting my hand on the stove because it hurts. The robot does not avoid running off the edge of the table because anything hurts.

    The Hard Problem is hard. Lots of very intelligent and clear-headed people—some of whom happen to be philosophers—have thought about it for millennia. Maybe it is not such a reasonable thing for you to think that you solved it in your spare time without putting any particular effort into even understanding what it is.

    ReplyDelete
  118. Sabine: "Personally, I would argue that consciousness is continuous and not binary (it's not on/off, it's small or much)"

    I take it you don't think electrons, tables, computers, and many parts of brains are conscious as that would be some form of panpsychism, so regardless how the consciousness of persons comes about, how does their being conscious but those other things not, square with your opinion that consciousness is "not on/off, it's small or much"?

    ReplyDelete
  119. So science claims sovereignty over the reality we call “ Love “ and files it away in the unsolvable department of the physics library under
    the “ feelings “ section . . No free will , no love . Easy - it’s a feeling , impossible to define and quantify but it’s our intellectual property and if we don’t have the answer then nobody does . Brilliant .
    You can only get away for so long telling somebody that the voice coming out of the radio is the radio communicating of its own free volition .




    ReplyDelete
  120. Sabine,

    And just for the record, your "refutation" even of panpsychism does not work. First: the claim is that even electrons have experiences, or proto-experiences, not that they can "think". Second, it does not follow from this hypothesis that different elections in the same quantum state have different properties: it is consistent that they all have the same experience. Third, the postulate that even different electrons in the same quantum state can have different further properties has not been experimentally refuted. Take Bohmian mechanics as an example. According to that theory, electrons have physical properties—precise positions—that are not reflected in the quantum state. These are the additional "hidden" variables in virtue of which the wavefunction is not complete. And electrons in identical quantum states (say the ground state of the hydrogen atom) can have different positions.

    But this theory which postulates the additional variables has never been empirically refuted. It makes the same predictions as standard quantum mechanics. And if you want to talk about quantum field theory, the the same point is made by Bell-type quantum field theories on a lattice. See Bell's paper "Beables for quantum field theory".

    I do not myself find panpsychism at all plausible. And I do not even think it solves the Hard Problem: if we could understand how the sorts of conscious experiences we have could result from the agglomeration of the proto-experiences of electrons and quarks, then I think we could understand how they can arise even from the agglomeration of electrons and quarks that have zero experience. That is, the transition from low-level proto-experiences to our experience seems not a whit easier to understand than the transition from low-level complete lack of experience to our experience. But picking holes in proposed solutions to the Hard Problem is like shooting fish in a barrel: your proposed solution just as much as the others. That's why the problem is hard.

    ReplyDelete
  121. Sabine
    You say "The reason you can't tell that other people aren't unconscious zombies faking consciousness is that you do not normally know the state of other people's brain"

    You're not claiming that one can refute solipsism by looking at other people's brain, are you?

    ReplyDelete
  122. So if I understand correctly, you can tell how many internal states an electron can possibly have by counting the number of electrons that are produced in particle collisions, and such counting rules out that electrons carry any more distinguishing properties than those we know about.

    Is this correct, or am I misinterpreting? The following paragraph confused me:

    "The third way is to assume that the existing particles are composed of more fundamental constituents, that are, however, so strongly bound together that we have not yet been able to tear them apart. With the third option it is indeed possible to add internal states to elementary particles."

    I wonder why the same argument (counting number of electrons that are produced) does not rule out internal states of the composite-particle kind? If answering this question would take up too much of your time, I understand of course!

    ReplyDelete
  123. Vincent,

    It does not rule them out - there are still variants of such models that are viable. But I don't see how these degrees of freedom would help you if what you want to explain is how consciousness in our own brains comes about.

    ReplyDelete
  124. @MAtT said how does their being conscious but those other things not, square with your opinion that consciousness is "not on/off, it's small or much"?

    I think the answer is simple enough; a fly with a million neurons can have a little consciousness, a human with a 100 billion can have a lot. Monkeys, dolphins, octopi, elephants are somewhere in the middle.

    ReplyDelete
  125. Hi Sabine.. Ok I get it. I will try to write a paper on it and pay attention to "poke poke" yes you can invoke qualia by manipulating brain directly.. I will keep that in mind. Thanks..

    ReplyDelete
  126. Tim,

    A brain is a lot of particles. A computer is also a lot of particles. You can learn from studying the former what kind of structure and activity gives rise to experiences and consciousness. Then you can try to find out whether the latter also displays it. It's exactly because the elements of both are emergent that it doesn't matter what they're made of. Also note, though, that I already said above that I don't think present-day computers will get us very far.

    Yes, you are correct in pointing out that supply of nutrients is not the same as neurons firing. I am sure that neurologists have noticed that tool.

    "I can design and build a little wheeled robot that avoids the edges of tables. I can understand completely in terms of physics how it works. But if you think that the little robot therefore feels pain..."

    You didn't follow what I said. First, I certainly do not claim that I know what pattern corresponds to an experience. I have repeated this like a dozen times.

    Second, I never said anything to the extent that a system that avoids certain things necessarily feels pain. In fact, whether the system does or does not manage to avoid something is entirely irrelevant. Even if you are entirely immobile, you may still feel pain. Point is that you have a system that projects an undesirable outcome under certain circumstances. And again, I am not claiming that this is the right notion of experience, it's just an idea I have toyed with. The point is simply that there is no reason to think there is anything about pain that cannot be explained in terms of brain patterns.

    "the claim is that even electrons have experiences, or proto-experiences, not that they can "think"..."

    If you do not change the standard model, what you have is the standard model. If you want to call that "panpsychism", fine, but that's both meaningless and useless. In that case all you have is the same old physics and explaining consciousness is as hard as it always was.

    "your proposed solution just as much as the others"

    Not sure what this means, but in any case I am not "proposing" a solution. I am not a neuroscientist and do not plan on becoming one. What I am telling you is that consciousness is no longer terrain of philosophy, it's science.

    ReplyDelete
  127. MAtT,

    I already explained this above. Consciousness, experience, thoughts, and so on are emergent properties. You need a system that has multiple interconnected parts so that you can use it to encode information about something else. I can't tell you just where the threshold is, but since elementary particles are qua definition the simplest things there are, you cannot use them to encode anything more complicated than themselves.

    A better understood example is maybe memory. To be able to store memory you need a system that can be in various different states and those states must be sufficiently long lived and so on. A single elementary particle just won't do.

    ReplyDelete
  128. "What I am telling you is that consciousness is no longer terrain of philosophy, it's science."

    Maybe this is the source both of your disdain and of your confusions.

    "philosophy" and "science" are not mutually exclusive categories and never have been.

    The Hard Problem, for example, was clearly explained by Leibniz (and others before him). Leibniz co-invented the calculus. Was he a "philosopher" or a "scientist". Both, of course.

    For what it is worth, he was also a panpsychist. Read the Monadology.

    Philosophers working on the problem of consciousness work hand-in-hand with both computer scientists and with neurobiologists. Many know a whole lot more than you do about both of those fields. Well-conducted philosophy in many areas—and this is a prime one—is completely continuous with various branches of science. That is certainly the case with philosophy of physics.

    Further,. many physicists have tried to invoke consciousness to solve problems in physics: for example Wigner's approach to the collapse of the wavefunction. Penrose has famously argued that human conscious abilities, together with Gödel's theorem, imply that physics is not computable. His argument is wrong, but the point is that is glides seamlessly between considerations we would call "physical", considerations we would call "mathematical" and considerations we would call "philosophical".

    So your whole framing of the issues depends on a false dichotomy: science vs. philosophy. They do not have disjoint domains, or "terrains".

    ReplyDelete
  129. To Steve Evans

    How the dice appear to roll is not the point. The superdeterministic model posits that, regardless of how they appear to us, the role of the dice is NOT random. You may call it fate, Intelligent Design or whatever. But, the approach leads to nonsense. If everything is predetermined then one can easily accept that it was predetermined by initial conditions that Romans build the first airports in 100 AD. The creation of such structures would have puzzled them but so what? Then, the "universe" predetermined that airplanes appear 2000 years later Or,perhaps the "universe"predetermined that the Neanderthals invent railroad tracks without the slight idea of what they are "for." The "coincidences" of superdeterminism lead to an endless number of such absurdities -- none of which you address.

    "Emergence" is another absurdity. Dr H cites lack of conductivity in a lone electron. Well, just where would it conduct a current? A physicist will someday claim that tennis can only be explained, like consciousness as an "emergent property." A single person (our electron) cannot play tennis but the game "emerges" when there is more than one! Then, the game "self-organizes" into more complex forms -- such as doubles. Eventually, if one hovers over Forest Hills, one can see that tennis has self-organized into enormous complexity with dozens of games played simultaneously -- all spawned from a single emergent game. AND, it is all governed by physical laws! Tennis will only emerge in certain "shells" and like electrons will not occupy an intermediate shell. Thus one can have two or four players but never five or six. Even in the greatest self-organized matches, players must be even-numbered. All that from emergence!

    If Dr. H's superdeterminism is correct than the universe is ordered in such a way as to avoid the sort of historical anachronisms I mention just above. And, as for emergence, it an excuse for saying "I don't know" and akin to magic.

    Dr H all but asserts that philosophy is dead. That appears, in science, to include logic.

    ReplyDelete
  130. Reimond wrote: My comment was not meant to be a complain about you

    I didn't say you were complaining about me. I said it looked like you were trying to incite Evans and you made a couple of misleading statements.

    My impression was that you were trying to be funny. But since you made a reference to one of my comments, I thought there was a chance you had an opinion on it, so I asked.

    ReplyDelete
  131. Sabine wrote: I don't see any reason why that consciousness or experience should depend on the material of the network.

    Good point.

    I've got to ask if you find panpsychism more interesting to talk about than string theory? :-)

    ReplyDelete
  132. Evans wrote: So it has been shown that panpsychic assumptions can contribute nothing to an explanation of consciousness

    That's what I've been saying all along, and I've been asking anyone to explain how panpsychism is useful even as a proposition. Even if we assume it's true, what are we supposed to do with it? Likewise, even if we assume there is a god, what are we supposed to do with it? (in the context of understanding nature)

    ReplyDelete
  133. Tim wrote: For what it is worth, [Leibniz] was also a panpsychist.

    I think you know it isn't worth anything. Leibniz and Newton believed in God (different versions), and that isn't worth anything either.

    Tim wrote: Many know a whole lot more than you do about both of those fields.

    What's your bottom line on panpsychism? What do *you* know about it? Everyone in this blog seems to agree that we haven't figured out everything about consciousness yet.

    When you ask about lobsters experiencing pain, it looks like you're trying to make a point, but what is the point?

    For what it's worth, I don't have a problem with philosophers working with scientists.

    ReplyDelete
  134. Lawrence Crowell wrote "Panpsychism reflects a desire to have our existence scripted into foundations." But Emergentism reflects our giving up in the completeness of our foundations.

    Stanford Encyclopedia of Philosophy: Emergent Properties
    "emergent entities (properties or substances) ‘arise’ out of more fundamental entities and yet are ‘novel’ or ‘irreducible’ with respect to them"

    ReplyDelete
  135. @Tim Maudlin says science vs. philosophy. They do not have disjoint domains, or "terrains".

    Philosophy allows and then expands upon and reasons with claims that have no physical evidence whatsoever and are also illogically argued; such as panpsychism. Traditionally, at least, physics has demanded experimental confirmation of speculations, which philosophy does not demand.

    Other than the common feature of initial speculation, those seem like disjoint domains to me.

    ReplyDelete
  136. @Steven Mason: "Does there necessarily have to be emotional content in a conscious experience? Do worms have emotional content?"

    That is the question being posed. @Tim Maudlin mentioned an edge-avoiding robot that reacts to inputs from its sensors with no more emotional content than a light switch. Robotics engineers have already made "worm" robots (and various other robotic insects) along those lines.

    How much of a worm is a "perfect" worm robot if its control system exactly mimics a biological worm's? Say it mimics eating and excreting, but takes no nourishment, does not break down its "food." Say it mimics mating, but produces no off-spring. But in all appearances and behaviors it is indistinguishable from a worm.

    Compare and contrast.

    Imagine a planet like Earth with robots matching every person on Earth. They perform, in real-time, exactly what happens on Earth (due to some signal they observe watching their "person") Short of examining their inner workings, Earth and its doppelgänger appear identical.

    But there is no emotional content whatsoever on the mirror world. Any more than our mirror reflection has its own inner life. Yet alien observers watching only the mirror world would see beings acting as though they had inner life.

    If this Robot World (actually David Chalmers' Zombie World) is a coherent idea, if we can imagine it as a possibility, then why do we have such a rich inner life? Why all this meta-thinking ability? What is the value of thinking about thinking about thinking about thinking about...

    Why do we love music?

    ReplyDelete
  137. [not sure if this is a double post. Please don’t approve this one if it is a repeat]

    Sabine,

    I think your intuition re: qualia is (almost) exactly right: “poke, poke”. I think it can be explained using terminology from semiotics which I haven’t yet seen in this discussion.

    I am going to make the following assumptions:
    1. “Consciousness” is about certain physical processes. A “conscious state” is a dynamic state with one or more conscious-type processes happening over and over.
    2. Any physical process can be diagramed as “Input —> [Mechanism or physical context] —> Output]”
    [see David Deutsch’s Constructor Theory for the basis of this assumption)
    3. Whether a process is a conscious-type process is determined by constraints placed on the Input, the Mechanism/Context, and/or the Output.

    I propose that a conscious-type process is a process as described in 2 above with the following constraints:
    a. the Input constitutes a symbolic sign (see semiotics), and
    b. the Mechanism is organized such that
    c. the Output is a valuable response relative to the reference of the sign introduced in part “a” above

    [I need to point out that “consciousness” would not be attributed to the mechanism described above. Consciousness would be attributed to the system which uses the mechanism].

    As a toy example, let’s consider light bouncing off a tiger, entering the retina. From there a chain of neural processing happens leading to a single neuron which then fires (remember, toy), sending a neurotransmitter to a mechanism. Under normal conditions, that neuron only fires in response to a tiger, but it fires in response to any sufficient viewing of a tiger. Let’s assume the mechanism is organized to have exactly one response to “tiger”, which is “run” (so, fire off signals to start the “run” process.). This system could be said to have had a “tiger” experience, but the system as described would not have any memory of said experience.

    Now let’s change the output so that instead the output is the creation of a unique pattern of neural firing in a specific “workspace”. That pattern in that workspace would constitute a memory of that “tiger” experience, i.e. the experience which created the pattern. Let’s assume there are other mechanisms which take input from that workspace. One mechanism may respond to that pattern in the workspace by running the “run” program. Another mechanism might respond to it by running the “shout ‘Ach! Hans! Run!’” program.

    So what is qualia? Qualia is a reference to a pattern, namely, the pattern which is the reference of the sign, in this case “tiger”. Is it a physical thing? No. Can we have the same physical events happen without generating that reference to “tiger”, i.e., that qualia? No. What we can do is “poke, poke” some neurons up to and including that last neuron, thus generating the experience which means “tiger” to the Mechanism.

    Can an electron have an experience? Only if you can identify an input which is a symbolic sign and a mechanism (within the electron?) that is organized to generate a valuable response relative to the meaning of that sign. The hard part might be explaining “value” for an electron, partly because the value of a mechanism is not usually attributed to the mechanism, but instead is attributed to that which created the mechanism.

    There. All clear.

    *
    [James of Seattle]

    ReplyDelete
  138. (Part 1) Tim, I understood your point and mentioned in my post that you may not agree with the solutions proposed thus far, but there are indeed a number of serious solutions proposed and being discussed pretty extensively in the literature. There are also a number of conferences each year that include the Hard Problem as a major focus, including The Science of Consciousness conference that I've attended most of the last ten years (in Tucson every other year and an int'l location in the off years).

    As for IIT's focus on integrated information, Tononi and his team explain this pretty well but I share your disagreement with the notion that it is integrated information itself that should be the seat of consciousness. My impression is that Tononi et al. are shifting toward focusing on causation rather than integrated information as the core concept for consciousness (Erik Hoel, a former postdoc of Tononi's focuses on this in his work and Tononi is hiring a postdoc currently who is to focus further on the philosophy of causation, as the link I sent shows). I've discussed a little with Tononi my thoughts on the need for a different notion of causation in his work and I've also been working on a different paper for a long while now that tries to add some cogent thoughts to the literature of causation and mind.

    Aaronson is mistaken in his critique of IIT with respect to the concern that a number of laptops could be considered conscious. This collection would be in IIT a "feed forward network" and would not have much phi at all as a collection, and thus shouldn't be considered a separate conscious entity. This is because Tononi's "exclusion principle" extinguishes subsidiary conscious entities in a collection once a higher phi entity is created through further integration of information. My 2014 paper "Taking Time Seriously in IIT" criticizes the exclusion principle as being ad hoc and I propose a Whiteheadian notion of time and the "creative advance" as a replacement for the exclusion principle in order to correct this problem with IIT.

    With respect to GRT, our general resonance theory of consciousness, we adopt the Whiteheadian notion of actual entities as the atoms of reality, and each actual entity is a "drop of experience." This isn't necessarily contradicting today’s physics, it's simply replacing the notion of "vacuous actuality," which is the assumption of today's prevailing physics, with a nonvacuous actuality, a necessarily experiential actuality at all levels of the nested hierarchy that is existence. Whitehead's 1929 book Process and Reality is the magnum opus for these ideas as I'm sure you know.

    So to answer your question "what really creates or is consciousness" we would say that all actual entities are conscious but include both mental and physical aspects as they oscillate from potentiality to actuality in each moment. All things resonate and thus all things sense ("prehend") the world around them and then become concrete ("concresce") based on those sensations.

    ReplyDelete
  139. (Part 2) Free will and mentality is located in that process of concrescence and is how the electron displays its little iota of mentality, as it decides where to manifest in each moment. That iota of experience, free will, consciousness, is magnified through each level of combination in the long nested hierarchy that forms each human being, and results in the rich consciousness and free will that you and I enjoy.

    This is what Freeman Dyson means when he writes: “[T]he processes of human consciousness differ only in degree but not in kind from the processes of choice between quantum states which we call ‘chance’ when made by electrons.” David Bohm wrote similarly in 1987: “Even the electron is informed with a certain level of mind.” So it's "choice, not chance" in QM, as I've suggested in my writings.

    Addressing your concern about evidence for panspychism I point you toward, if we are looking at the base of the ontological chain, Dyson's and other comments that interpret the probabilistic results of experiments like the two slit experiment as demonstrating choice, not chance. This is evidence of mentality.

    And if we start at the top of the scale, we gather good evidence every day when we observe the various behavioral correlates of consciousness (BCC) in other humans (you, for example, seem pretty conscious to me even though we've only interacted via words on my computer screen, because you demonstrate behavior that seems pretty similar to my own). We look to other animals and use the same test. My cats seem pretty conscious because they demonstrate many BCC that are quite similar to my own and other humans. Ditto with rats, bats, gnats and even down to bacteria that show communal hunting behavior, avoidance behavior, etc. for example. So we have a universe of evidence for consciousness. It is a bit of a backwards way of thinking, it seems to me, to assume that only humans or other mammals are conscious when we have so much evidence of mind all around us.

    I'm camping this weekend but will check out your paper when I'm back in the saddle this week.

    ReplyDelete
  140. Dr. A.M. Castaldo wrote to me:

    >[Dave said]: ...and what sort of substrates in general can serve as a basis for consciousness. Those are real, and so far unanswered, questions.

    >[AMC replied]:They are not unanswered! Asking this question presupposes that the substrate we see, neurons and other biological mechanisms in the brain, are an insufficient basis for consciousness.

    Well... I thought I had made clear that my question related to whether machine consciousness (i.e., in silicon chips) is possible. Lots of people think so, lots of people think not. (I myself lean towards the skeptics.) Seems to me an unanswered question. If you do have the definitive answer to that question, I guess I missed your explaining it. Sorry.

    AMC also wrote:
    >The real questions are answered, there are just a super-majority of people that don't like the answers or their implications, and thus are constantly inventing new fantasies to try and avoid the real answers.

    Well, perhaps if, as you say, a "super-majority" reject your answers, then you have yet to express your answers in a sufficiently clear and convincing manner.

    I do find intriguing the fact that most people on both sides are absolutely convinced that their own views on this issue are obviously true and yet they are unable to make them obvious to people with opposing views.

    ReplyDelete
  141. @ Jim Peterson re: anesthesia

    I was unaware until recently of the various stages/planes of anesthetization.

    I was having a minor procedure done, involving a large needle near the spine, so I elected for general anesthesia thinking it best that I be "immobilized".

    The first time it was done, I was in the staging area and, the next thing I knew, I was back in the staging area.

    The second time, they asked how the first had gone and, since I had no issues, indicated they would make no adjustments.

    The second time, I was "awake" enough to transfer myself from the gurney to the table, roll over, position myself, etc. At some point, I "went under", or so I believed.

    I mentioned the difference in experiences to a nurse, and it was at that point that she explained that, actually, I was "awake" throughout the procedure, ostensibly not immobilized, but rather only rendered amnesic, all by design.

    I was shocked. The last thing I wanted was to be talking bullshit while they worked.

    In fact, I vaguely recall telling a nurse (a female, I hope) during the first procedure, as she shaved my back, which felt good:

    "You can keep going."

    No more surgery for me!

    R. Taylor

    ReplyDelete
  142. I must thank The Queen for not booting me off, and her and everyone else for the opportunity to be "in the room" with such a sharp bunch.

    I am a lay-person (and I don't use that term loosely... no offense to prostitutes), but have long-pondered the difficult questions about our existence in the cosmos, as well as the cosmos themselves.

    Twice now (and I only discovered Bee's blog two days ago), I have run across a comment by another that just stopped me in my tracks due to its serendipitous connection with some postulate or thought of my own.

    I must point out that I am without question the least-educated to darken this comments section, and that is no idle boast. As evidence, I will note that, when I was in the 8th grade, our school system offered a deal by which a student could foreclose any requirement for math courses in high school by merely passing algebra in the 9th grade. My only question was:

    Where do I sign?"

    I've already addressed the first comment that made me do a double take; the second is one by "James of Seattle". James said:

    "You will always have a bottom level, a level which we have not observed any sub-parts. At this level, it makes no sense to try to ascribe a mechanism, because there would be no way to differentiate that mechanism from angels."
    (emphasis added)

    The "angels" reference caught my eye, because I had only the day before read an article that might be tangentially related to this discussion, if not Bee's chosen area of research, and found a passage which seemed to me to amount to the invocation of... angels! I am not a religious person... though I would never claim to "know" we have no Creator... but the description of the mechanism proposed in this article seemed positively supernatural:

    " ...the bendy space-time fabric in its interior is “holographically dual” to a quantum theory of particles living on the lower-dimensional, gravity-free boundary."
    (emphasis added)

    I'm sorry, but I read that and thought: 'Angels! A lot of them! The article:

    https://www.quantamagazine.org/how-space-and-time-could-be-a-quantum-error-correcting-code-20190103/

    R. Taylor

    ReplyDelete
  143. Uh-oh!

    Bee says:

    " ... how do you know that machines do not experience anything? They probably do, just not very much."

    Isn't that at least incrementally removed from "Electrons don't think?" : )

    ReplyDelete
  144. The Nirvana Shatakam of the Adi Sankaracharya begins:

    mano buddhi ahankara chittani naaham

    and the fourth line is

    chidananda rupah shivo'ham shivo'ham

    ---
    The English translation (roughly) of the first line is

    I am not the mind, the intellect, the memory, the ego or the memory

    and the translation of the fourth line is

    I am the form of consciousness and bliss, I am the eternal Shiva...

    ----
    Why do I bring this up? Because in this philosophy, however mistaken it may be, the content of consciousness is not "inner life, thoughts, mind, etc.". So what is this "consciousness" talked about in the Nirvana Shatakam? Perhaps it has no proper English translation, but "consciousness" is the word that all the translator scholars use. In deep meditation, when thought slows down, the space between thoughts offers a glimpse of this consciousness, we are told. So the point is that you have to understand what precisely these philosophers mean by "consciousness" before you can raise the true objection "electrons can't think, etc." because it may be an irrelevant objection.




    ReplyDelete
  145. Unknown,

    No, this Dr H does not "assert that philosophy is dead". This assertion was made by a different Dr H (who, alas, is dead).

    ReplyDelete
  146. Stephen,

    "I've got to ask if you find panpsychism more interesting to talk about than string theory? :-)"

    Certainly my readers do...

    ReplyDelete
  147. Tam,

    "Free will and mentality is located in that process of concrescence and is how the electron displays its little iota of mentality, as it decides where to manifest in each moment. That iota of experience, free will, consciousness, is magnified through each level of combination in the long nested hierarchy that forms each human being, and results in the rich consciousness and free will that you and I enjoy. "

    I'm afraid you still did not understand what I say. Unless you change something about the electron (or other particles) there are no "iotas" that can be "magnified through each level of combination". If you do not change the standard model, you have physics as we know and like it and that leaves explaining consciousness as hard as it has always been.

    ReplyDelete
  148. Quentin,

    "You're not claiming that one can refute solipsism by looking at other people's brain, are you?"

    No, as I already said above, solipsism is unfalsifiable. It's also a pretty useless philosophy. Whether or not you believe that your observations are real, the assumption of an external reality works very well to explain what you observe. The only difference is that the realist would say "it is so and so" whereas a solipsist would say "in my model of the world it is so and so". For the scientist, it doesn't make a difference.

    In any case, the problem with the philosophical zombies isn't identical to the solipsism problem. Even if you're a realist you still have the zombie issue. I am telling you that if you manage to identify brain patterns (for the solipsist: models of brain patterns) with experiences (models of experiences) you can decide whether someone has experiences provided you can observe their brain. That's a trivial collorary.

    Somewhat more interesting is the question whether someone (something?) that does not have experiences can behave in a way indistinguishable from someone with experiences. I think the answer is yes (works like the Chinese room) but it's exceedingly unlikely to happen as a product of evolution (no models, no predictive power, too computationally intensive).

    ReplyDelete
  149. Lawrence Crowell states:

    "There is a long history on how these types of ideas end up on the trash heap, even if some still cling to them long after their demise.

    Do we really think that we are done with ideas/theories... even those in mathematics-based fields such as physics... ending up on the trash-heap? If so, it will be a first.

    I took just enough Engineering courses to get me into trouble (after the math-free stint in high school), but have had, even prior to those limited courses, a practical engineering ability second to none. Bee has car trouble; I have a 1974-model car I've had for over forty years, and a 1965-model truck I've had for over twenty years, both because I allow no one else to work on them (especially physicists : )

    One belief was nurtured during those courses: There is no problem that engineering can't solve. But of course, that only holds true in those domains that are the province of engineering.

    Much of what the field of physics has shown us is unlikely to change but, from a distance, this would appear to be some of the more elementary posits of the field. The strictly theoretical and highly esoteric areas are sure to see as much if not more revision than has been the case in the past.

    It seems to me that, until we can go to the edge of the Universe (if there is an edge) and look back, until we can travel into the past and the future, we are dealing with a rather narrow range of observables and possible states of existence.

    Particle physics seems to be on fairly solid ground. But what does that tell us about the big picture? If we are working with a "particle horizon" that is defined as "a sphere whose radius equals the distance to the most distant object we can see", doesn't that place us dead in the middle of it all? Didn't that idea end up on the "trash-heap" long ago? And are we done "seeing" exponentially further with every leap in technology? How do we know we're not in some dark corner of the Universe, and yet still unable to see even the damned corner?

    I have said that if there were some entity capable of creating everything we know, it would certainly have been able to leave a trail of breadcrumbs designed to test our faith; could those breadcrumbs be mathematics?

    Pascal said, speaking to Bee (and I'm only assuming his characterization is correct):

    " ...or (as I think you have argued yourself) why do the physical constants have the value that they have."

    Could the seeming elegance and validity of mathematics be an intentional effort to tempt us to say:

    "See! 2+2=4, so there is no need for a Creator!

    In short, I believe we are "in the dark" re: consciousness just as much as we are re: the reason for the existence of everything, particle physics notwithstanding.

    ReplyDelete
  150. Unknown said...

    "How the dice appear to roll is not the point. The superdeterministic model posits that...."

    I don't see how this is an argument against SH's very specific point. It has been shown that it is physically impossible that panpsychism can be used to explain consciousness. Do you have an argument against this?

    ReplyDelete
  151. If you feel that scientists are superior to philosophers because of experiments and advanced math then do that rather than arguing. Take the idea of a particle soul seriously and do an experiment and don't prejudge. If a very large mass rare particle is a soul how would it interface with the brain -- probably electromagnetically since the the brain is electrical. It would have to have a lot of bandwidth to receive visual and audio information and other info in and detailed commands out. One way way to do this is have a lot of electrons surrounding it to receive precise frequencies as electron jump to higher and lower orbits.

    The particle would increasing be able to see and hear and be able to think as it matures because it inherited these abilities from its parent -- the universe which is a fully grown up conscious particle with free will.

    If particle souls are real it would be the greatest discovery ever, we could learn the interface code the soul has and transfer it to a custom designed body we could engineer for almost any environment in the universe and death and pain will be mostly a thing of the past -- a utopia.

    The experiments might even be cheap -- just pay attention to very specific frequencies the brain generates in response to very specific frequencies of light and sound and other stimuli and movements that might result in breaking the soul code. The location could be pinpointed by masers sending tones or colors after the code is cracked.

    It might sound far fetched but it would be the greatest discovery ever if found so I think it really is worth looking for homunculi.

    ReplyDelete
  152. Sabine,

    Sorry but I think we're going round in circle. Every time you explain how we could know someone has experience, I'm thinking: ok, but with the same method, we can know *what kind* of experience this person has as well. And every time you explain why it's an intractable problem to know what another person experience, I'm thinking: ok but if we cannot know *how* s/he feels, we cannot know whether s/he feels anything at all (or to say it differently, this unknown experience could as well be a void experience).

    So I'm still not convinced but it's probably not very useful to go round one more circle at this point, I'll stop there.

    ReplyDelete
  153. Kevin,

    We know what kind of particles there are, what properties they have, and how they interact. A "very large mass rare particle" that is "a soul" and "interface[s] with the brain -- probably electromagnetically since the the brain is electrical" has long been ruled out.

    In my book, I explain what I call "the hidden rules of physics" that are the methods that physicists use to invent new particles (or, more generally, new effects) so that the new hypothesis is still compatible with all existing data. It's not easy because there is so much data and the existing theories work so well already. It's really hard to change anything about them without running into conflict with experiment. Most people don't seem to realize that.

    ReplyDelete
  154. I feel it is important to go past "experience" and delve into "qualia" in order to take a scientific approach to exploring the nature of consciousness.

    Part of our problem with the word "consciousness" is the lack of a well defined common language. You could say the right words are not embodied in enough brains to have a meaningful conversation. And those words correlate with particular worldviews or perspectives, how our brain has parsed the massive stream of information coming in to construct our mental world. We end up talking past each other.

    I find the word "qualia" useful as a starting point to a reductionist exploration of consciousness. I see my conscious experience as a composite of fundamental building blocks, a kind of language that my body uses to communicate with the conscious part of me. My assumption here is that a reductionist analysis of consciousness will correlate with a reductionist analysis of the body in some way. It seems like a reasonable way to tackle the "hard problem of consciousness", something actionable instead of just talk. I don't assume that the correlation will be at any particular level. Hopefully the correlation will emerge from the information structure once it is organized logically.

    I don't see how we can have a very meaningful argument relative to emergence vs pansychism until we see the level at which there is a correlation between qualia and physical structure. It could be in the neural network. It could be in intracellular structures. It could be molecular. It could be at the atomic level somehow, but I don't have a clue how. There just isn't room for it in the standard model, unless it is somehow nonlocal, if that even makes sense.

    To me, consciousness is part of a control system. I find it most productive to study it in that context. My brain can control my body quite well without any need for consciousness, for the most part. Consciousness seems only necessary for handling the indeterminate, the undecideable, for navigating a paradoxical possibility space where there are no right answers. I guess I mostly believe it is an emergent property of that space, what it feels like to be in superposition.

    ReplyDelete
  155. Quentin,

    Sorry, I'll try it one more time. Maybe it helps sticking with what is (in principle) practically doable.

    Suppose you have a group of test people (possibly including yourself). You have a way to track what's going on in their brain in great detail. You put them in a large variety of situations in which you either know what they experience because it comes with the situation (say, free fall), or you ask them. You use this data to test your hypothesis about what kind of pattern makes an experience (the "hard problem" being to come up with a useful hypothesis to begin with). If you can reliably answer the question what the test subject experiences from the data of brain patterns only, you have an empirically established method of figuring out whether someone has an experience and also what kind of experience. (As I mentioned, people are already doing this.)

    That would not enable you to tell, however, whether someone else's brain experiences something the same way as you do. The best you may do is to say that since your brains are similar in many regards, it's probably the case. But there is no way you can ever test this hypothesis. I think the real issue here is that the question doesn't make sense. It's like people who give me advice starting with "If I was you..." which, well, they are not. And if they were me, they'd do what I do, so really what sense does this make?

    Now, if you're a solipsist you may want to deny that any of those other people even exist and also the whole experiment is really just your imagination and so is, presumably, any reading of your own brain activity because, heaven knows, maybe you are a disembodied cloud and don't actually have a brain. This is all very interesting speculation, but since it's unfalsifiable it's also rather pointless. In this case you may want to stick with saying something to the extent that your observation is well-described by the hypothesis that you have a brain and that there are other people who have brains that are similar to yours and that those people have experiences similar to yours and so on, which fapp has the same consequences.

    Best,

    B.

    ReplyDelete
  156. @PhsyicistDave said: I do find intriguing the fact that most people on both sides are absolutely convinced that their own views on this issue are obviously true and yet they are unable to make them obvious to people with opposing views.

    There is a simple answer to that too. The problem is not making one's views obvious, the problem is the stakes for the debaters are so low. There will be no cost to the people with opposing views if they are ultimately proven wrong. Nothing depends on being right. Philosophers won't be punished, their academic culture doesn't punish them for pursuing ideas that rely on unprovable elements. From what I have read of university philosophers, they love that stuff, and to me seem disdainful of simple mechanical or biological explanations.

    FWIW I did explain my position above, I think it is clear. I am not absolutely convinced, I outlined a hypothesis of consciousness and qualia consistent with the evidence from repeatable experiments I know about, and relies only on the known biological mechanics of the brain; it doesn't rely on QM or any supernatural elements or hypothetical physics (like panpsychism). Heck, I could teach it to Freshmen.

    I put my view out there but don't expect many people to adopt it. Not because it is wrong, but because it doesn't cost them anything to believe whatever they want to believe, and to them believing that consciousness is magical, or mysterious, or special, or relies on a new mystery force that brings back paganism and imbues all things with "spirit" -- Well I think that's more fun for them. I'd rather know the truth, good or bad, but I know I'm in the minority.

    ReplyDelete
  157. Biologists and neurologists appear to have made progress in understanding the function of neurons. Also some progress in understanding how they connect together to make biological brains.

    This gives me hope that if they keep plugging away, eventually they will have worked out all the details of how brains are built.

    When this done,however long it takes, it seems to me we will be an annalogous situation to someone familiar with electrical components such as resistors, capacitors and transistors who owns a radio set but doesn't understand how it works. By examining the circuitry in detail it would be possible to reverse engineer your own radio by copying the details of the one you have. By building artificial neurones and connecting up in the same way as the real brain we should in principle at least be able to reverse engineer a mechanical brain machine.

    If we do this, if you believe consciousness emerges from the wiring of the brain and requires no fairy dust, the artificial brain should exhibit it.

    It seems to me that reverse engineering and building a artificial brain may be anoint way off but it is at least a practical and tractable way forward. And therefore all the philosophical meanderings seem at best premature when there is a practical possibility of an empirical test.

    We just need to put the speculations on hold and await the day when neurolgy and engineering get to the point we can build an actual brain. It is clearly possible because nature has already done it.

    David Millier
    London UK

    ReplyDelete
  158. @ R. Taylor: I don't think you read or understood my statement or this piece I wrote. I was referring to anthropocentric ideas. These pretty clearly have been discredited. The cosmological horizon or particle horizon is frame dependent. Every observer in the universe would conclude they are at the center of a region bounded by this horizon. Also the spacetime of this cosmology or observable universe has to edge. It might be finite, but with a spherical or toroidal topology.

    @ Tim Mauldin: Gödel's theorem might have something to do with physics, in particular with quantum interpretations. Roger Penrose was on to something I think, but he got derailed a bit. If one is going to think about these things one needs to be very careful and conservative. Also one needs to keep these things as ideas to play with and not deeply held. A quantum measurement is where a macroscopic system, ultimately composed of many quantum states, couples to a prepared or post scattered or tweaked system with few quantum states. This is the system to be measured. The quantum phase for the superposition or entanglement of the measured system enters into the macroscopic system. This is FAPP an intractable problem. We might see this as a self-referential loop. We are left with this open question on what is meant by measurement and whether there is some dynamics or physics that predicts or describes that. So far no dice, and we only have lots of conflicting interpretations. This has features at least similar to ω-inconsistency of a formal system that is complete.

    There was the recent result of Frauchiger and Renner on how an observers witnessing the accounts of primary observers can contradict each other. The argument is really very similar to Bell's. The result is that quantum physics does not account for macroscopic systems, which means the Bohr Copenhagen Interpretation is given a boost, and MWI is deflated. Quanta defy ordinary existential categories and it is not possible to have ψ-epistemic interpretations (Bohr's CI, qubism etc) that agree nor is is possible to have ψ-ontic interpretations (MWI, Bohm, consistent histories etc) that agree. These also do not agree with each other. I also do not think it is possible to logically conclude from the postulates of QM whether quantum mechanics is ψ-epistemic or ψ-ontic. We have no decision procedure for knowing which of these is correct, which is similar to the negation of the fifth axiom of Euclid which gives you lots of different geometries. “Shut up and calculate” ideology of Mermin is maybe similar to saying “this system is incomplete” (ω-incomplete) and to avoid these inconsistencies don't try to make it so with interpretations. So Gödel's theorem may well be lurking here and represent some limitation on physics.

    @Phillip Thrift: Emergent systems are based on structures that are thought of as more primary. Spacetime may be emergent from large N entanglements of quantum states. Hence spacetime, which we can only define according to rulers and clocks with little deeper existential insight, may be then emergent from quantum states which are even more existentially uncertain. We can chase the dragon down the hole into the vacuum and when we bit its tail we find we have a sort of cotton candy of nothingness in our mouths. There is a sort of middle ground in this chain of emergence, with things like planets, stars, rocks etc that are fairly hard and appear to have a certainty of existence. Then we continue up the chain of emergence and complexity where we ponder consciousness, and we curiously find ourselves again chasing after nothingness.

    ReplyDelete
  159. Sabine,

    Thank you I understand better. I think we were using "experience" differently.

    What you describe in the first paragraph looks like what some neuroscientists and philosophers of mind call "finding the neural correlates of consciousness". This is not the "hard problem" that Chalmers describes.
    The intractable problem you describe in the second paragraph looks very much like what Chalmers calls the "hard problem of consciousness".

    The problem is: no piece of physics will ever tell us *what it's like* to have such or such experience (as you accept). For what we know it could as well be like *nothing*, and physics wouldn't be different. Yet we know that it feels like something to have experiences.
    Conclusion of the argument: physics must be somehow incomplete; it tells us what things *do* but not what it's like to *be* something.

    There are people who reject this argument for various reasons and others who accept it.
    If you accept it, you can be like "it's a question with no answer why worry?" (which seems to be your stance). Or you can try to answer it by embedding our physical knowledge into a metaphysical picture that contains "qualitative feels" and have metaphysical theories about it.
    And if you choose this option, then one possibility is to be a panpsychist, that is, claim that the "feel" is something widespread and fundamental in reality, and that our own "feels" are but particular (presumably highly structured) instances of it.

    All this to say: if you understand what the debate is and what panpsychism is after, the objection that if electrons had thoughts, that would change the physics, is irrelevant.


    ReplyDelete
  160. Quentin,

    Yes and no.

    It is right that I personally think it's pretty useless to worry about questions that are unanswerable. But I don't have a big problem with people who want to spend time on those. I do get a problem, however, if they end up claiming something that's demonstrably in conflict with science. You write:

    "the objection that if electrons had thoughts, that would change the physics, is irrelevant"

    No, it's not. Because if it changes the physics it'll end up being in conflict with experiment, so it's de facto wrong. And if you make it so that it does not change the physics, then you cannot explain anything with it. The best you can do is come up with a consistent belief-based interpretation.

    Now, if you want to do that, introduce panpsychism as a belief-system that by construction does not have observational relevance or explanatory power, that's fine with me. But then you should be very clear and upfront about it and not pretend otherwise.

    (And, frankly, I don't know how this would count as philosophy, to me it looks like religion.)

    You write:

    "What you describe in the first paragraph looks like what some neuroscientists and philosophers of mind call "finding the neural correlates of consciousness". This is not the "hard problem" that Chalmers describes."

    In response, let me quote from scholarpedia:

    "The hard problem of consciousness (Chalmers 1995) is the problem of explaining the relationship between physical phenomena, such as brain processes, and experience (i.e., phenomenal consciousness, or mental states/events with phenomenal qualities or qualia)."

    Finding this relationship is possible by using the scientific method, and it's a question that is perfectly answerable.

    "The problem is: no piece of physics will ever tell us *what it's like* to have such or such experience (as you accept)."

    This statement doesn't make any sense to me. If you have an experience, then that will tell you what it's like to have this experience, and that experience is all physics. It's correct that you will not be able to tell what it's like to have experience X by looking at (say) a computer image of someone having experience X. That's because you looking at a brain pattern isn't the same as your brain having the same pattern. (That's the confusion underlying Mary's room.) If your brain and the other person's brain are sufficiently similar, you may be able to try and copy their experience. But of course you'll never know for sure that you indeed feel the same.

    ReplyDelete
  161. David Millier said

    "It seems to me that reverse engineering and building a artificial brain may be anoint way off but it is at least a practical and tractable way forward. And therefore all the philosophical meanderings seem at best premature when there is a practical possibility of an empirical test. "

    Unfortunately this experiment may well be a very long way off, not least because if the physicist, Roger Penrose's idea if true, it may be necessary to consider microtubules as operating as some sort of quantum computer, and there are many inside every cell!

    That will require one giant computer simulation, which isn't going to happen soon, but at least it can enter our discussion as a gedanken experiment.

    There is presumably no need to imagine this at the transistor level, but simply at the computer level, which is much more familiar to most of us.

    This gedanken experiment has given us a fully computational model for consciousness, which is what Sabine has already claimed is correct, so let's try to derive something of a contradiction from this idea.

    Suppose the simulation starts with a snapshot of a human brain sitting still and contemplating something awful - the break up of a love affair, a road traffic accident, or whatever. The real human being simply sits there and suffers as he contemplates this.

    The computer will start with a simulation program P and a large quantity of data representing the brain snapshot D. We turn on the computer simulation and the program either doesn't generate any output (because the actual human is just sitting thinking) or it delivers the new brain state, D1 after the simulation is complete.

    We can represent this symbolically as:

    P+D => D1

    Since all the operations of the computer are presumably deterministic (any random numbers required are generated using pseudo-random sequences) the 'equation' P+D => D1 is actually a theorem - you only need the computer to check the theorem!

    Thus if you assert that this simulation suffers in the same way as the human, then that suffering isn't anything to do with the brain, or computer, it belongs in a book of theorems!

    ReplyDelete
  162. David Miller

    Do we know the principles that created the brain in the first place ? cargo cult plane me thinks is all we are looking at here .

    This is nothing more than the reduction of knowledge in its many forms to only one technique, namely scientific knowledge. Sabine expressed this perfectly in her unsolvable statement and the premise that science alone is the only technique capable of unlocking answers and if we can’t answer the “unsolvable “ with scientific technique then it’s pretty useless even raising the question ! Ultimately this position leads to scientific technique being the sole shaper of reality and ultimately truth. Obviously that is science eating it’s own .







    ReplyDelete
  163. Everyone keeps trying to convince everyone else of... something, and the conversation has gone very circular and isn't going anywhere.

    If I'm following (and it's very possible I'm not), as a physicalist (and instrumentalist, to boot), Sabine sees the notorious "hard problem" as an engineering problem involving our ability to fully understand how the brain works.

    Get good tools, study the right bits long enough to see the patterns, develop a theory (test it), and we should understand everything there is to know about the "mystery" of experience. Mary can fully understand color from within her room.

    For instance, Mary could develop a tool that implants in her mind the neural correlates she knows will cause her brain to "experience" redness. (Just consider the movie, The Matrix.)

    But this is not the hard problem. This is just an engineering problem. One we will solve.

    The hard problem, as the scholarpedia quote suggests, is explaining the relationship between these neural correlates and Mary's subjective experience with redness. (Which she can experience either through actually looking at something red, or by having her tools manipulate her mind. Same thing.) It is a bit of a "why" problem, a philosophical problem.

    Consider Mary a bio-machine for detecting red. If I follow, Sabine is saying Mary's "experience" of detecting red is the machine's response for "I've detected red." In a very simple machine, a photocell with a red filter, a buzzer sounds for redness. In the Mary-machine, she "experiences" it.

    Others (full disclosure, including me) believe there may be a deeper mystery behind the subjective experience. If so there is a good chance that mystery involves metaphysics of some kind. As someone with dualist leanings, I'm okay with that, I like living in a universe with a little mystery and metaphysics.

    But physicalists, such as Sabine, reject that view entirely, which I think explains why the "hard problem" is a hard sell. Maybe we should just accept there are different perspectives on this matter.

    ReplyDelete
  164. Sabine,

    Let it be clear for once: panpsychism is not and never pretended to be an attempt to change physics.

    Panpsychism is not a *physical* or causal explanation of anything. But according to its defenders, it is a *metaphysical* explanation of why we have qualitative experience ("feels like"). Again, that's a huge philosophical literature on what counts as an explanation. Perhaps some would side with you but some believe that such metaphysical explanations make sense.

    You misinterpret the citation from scholarpedia. The hard problem is not to *find* the relationship. It is to *explain* it. Chalmers is not an idiot: of course he knows that finding the relation (neural correlates or whatever) is empirically tractable, that's precisely his point.
    He is very explicit that it's *not* the hard problem. Believe me or read the literature (Chalmers or secondary) if you don't.

    In the last paragraph, it seems to me you're merely re-saying what I'm saying. So yeah I agree...

    ReplyDelete
  165. Let's try another way to say this:
    Panpsychism is exactly, in your words, a "belief system that does not have any observational relevance nor explanatory power" -- at least so long as you apply it to account for external phenomena. It is idle. But when you apply it to *yourself*, then it explains that you feel something, that there's something it's like to be *yourself*.

    ReplyDelete
  166. @Quentin: Chalmers problem (using the scholarpedia definition Sabine provides) is really not that hard! In fact I think it is based upon a misunderstanding, because the "experience" of consciousness and qualia are physical phenomena and brain processes, they are not separate from the brain. Chalmers is implicitly asserting that they are different, without proof. The notion they are separate is an illusion, and there is no evidence or valid logic to support the assertions that consciousness is anything but.

    The neural correlates of consciousness in a real sense may not exist because the entire brain may be involved in consciousness, so there is no particular part to "correlate". Or you can flip that, the neural correlates pervade the entire brain and cannot be separately identified.

    Chalmer's "hard problem" is dismissed out of hand as a silly misunderstanding that some "relationship" must be defined between physical phenomenon, such as brain processes, and other brain processes such as consciousness and qualia and emotions and pain. No "relationship" is necessary!

    What would be necessary is for somebody to prove the assertions that the latter brain processes are not brain processes but something independent of the brain. Which to me is what panpsychism asserts, without any proof whatsoever.

    ReplyDelete
  167. Sabine.. You are not grasping the significance of Chalmers'"gap". Even if you map every neuronal activity in the entire brain you will never explain why the color red (to take but one example) looks as it does to the subject.That is his point

    ReplyDelete
  168. Postulating that consciousness is a quality shared at a fundamental level so that it arises first in fundamental particles and is somehow encoded within quantum properties seems to be overlooking a much more fundamental aspect of existence...awareness. To be aware is not the same as being conscious, though, at first glance the two appear to be homonyms. Consciousness demands inclusion of the notion of self-awareness; while awareness, by itself, only recognizes the practical solutions for reaction based on the immediate environment.
    In other words, self-awareness dictates the responses that must be made to sustain existence, hunger and flight immediately come to mind. While awareness by itself, only recognizes environmental constraints. Together, the two form a whole that begins the path to the quality we call Consciousness, and that, in my opinion, is only a property of life!
    Defined in this way it’s clear that elementary particles are aware of their environment in some fundamental way. ie. An electron “feels” the electromagnetic fields. A quark “knows” it’s confinement. And speculating that our own consciousness uses these qualities in some profound computational sense seems reasonable. But, in my opinion, any more esoteric theory borders on absurdity.

    ReplyDelete
  169. She says, "If a philosopher starts speaking about elementary particles, run."

    Well I say, "If someone without even a limited understanding of the philosophy of mind starts speaking about consciousness, run."

    I mean, seriously, you state, and I'm paraphrasing, that if an elementary particle is conscious, we would expect it (at the very least) to be able to change. That's a pretty big assumption you are making about consciousness. Consciousness, that is, awareness itself, is not necessarily tied to 'thinking' or 'mental processes'. Perhaps it is, but the point of panpsychism is that it is not necessarily so.

    The fact of the matter is, today's most recent scientific and philosophical understanding of the mind cannot explain how we get from raw neuro-processes to subjective experience - that is, how neurotransmitters such as dopamine translate into the subjective sensations of pleasure.

    Panpsychism offers a solution. Instead of assuming that consciousness is the product of processes within the brain, perhaps consciousness precedes processes within the brain, whereby the brain is simply a thinking, self-reflective, ego of a thing.

    Both these scenarios are equally plausible given our current understanding of the mind. The only reason we so readily subscribe to one and not the other is due to our dogmatic predispositions.

    ReplyDelete
  170. typo --- I wrote so the spacetime of this cosmology or observable universe has
    to edge
    The bolded to should be no.

    ReplyDelete
  171. Unknown wrote: The superdeterministic model posits that, regardless of how they appear to us, the role of the dice is NOT random. You may call it fate, Intelligent Design or whatever. But, the approach leads to nonsense.

    First, there's no need for "super." "Deterministic" works just fine. If an outcome is determined, saying it's superdetermined adds no additional information.

    Yes, the roll of dice isn't random. But for humans rolling dice, the results appear to be random because it isn't predictable. Again there is no need to invoke supernatural forces or an intelligent designer.

    Consider a gas. Even though we refer to the motions and collisions of the gas molecules as random, they are not random. We have models that predict the general behavior of the gas. If we had sufficient information and computing power, we could start to take some of the randomness out of the randomness.

    Consider loaded dice. Loaded dice, just like regular dice, are following deterministic factors. The key difference is that we have a sufficient amount of information about the loaded dice to make predictions. With loaded dice, the deterministic factors are more obvious to us.

    Consider gambling casinos. Casinos discourage card counting because it takes some of the randomness out of the games.

    When something is sufficiently complex, deterministic outcomes appear to be random to an unaided human observer.

    Unknown wrote: the "universe" predetermined that airplanes appear 2000 years later

    If we examine all the factors that resulted in you being born, it would blow your mind. Even if we go back a mere 5,000 years it would still blow your mind. The chain of your ancestry is dependent on countless factors that could not be predicted, and yet, here you are. It's amazing and it's not amazing.

    The universe is full of equally amazing things, such as airplanes appearing at a certain time and place. If dinosaurs hadn't gone extinct when they did, perhaps there never would have been a human species.

    It's hard for the human brain to wrap itself around all the deterministic factors at play in the universe. Also, most people simply don't like the idea of determinism. As far as I can tell, they think it takes away some of our "specialness."

    I'm an atheist now, but I was raised Catholic. Christians like to say that people can choose between good and evil, but if God is omniscient, he already knows who is going to heaven or hell. That raised all sorts of questions about determinism.

    More recently, I've heard Christians invoke determinism in the unfolding of the universe and evolution. God created the initial conditions and set everything in motion. In this way, science and religion are reconciled, or so they say.

    Unknown wrote: "Emergence" is another absurdity.

    Are you the same Unknown who argued that emergence was absurd last month? If so, why are you bringing it up again?

    Unknown wrote: A physicist will someday claim that tennis can only be explained, like consciousness as an "emergent property."

    You're willing to make absurd statements about emergence just to show how absurd you think it is. Interesting strategy. :-)

    ReplyDelete
  172. Sabine wrote: It's like people who give me advice starting with "If I was you..." which, well, they are not. And if they were me, they'd do what I do, so really what sense does this make?

    FYI: "If I was you" is an idiom, and as such is not meant to be taken literally. "If I was you" translates to:

    "If I was in your position, this is what I would do."

    Now, sure, you could ask why people don't just say "If I was in your position." We all have idioms that we like and dislike. I suspect people dislike this particular idiom because, as you say, it's something people say when they're giving advice. :-)

    I'm guessing that German culture doesn't use this idiom?

    ReplyDelete
  173. Chris wrote: in all appearances and behaviors it is indistinguishable from a worm

    This reminds me of the novel "Do Androids Dream of Electric Sheep?" (which the movie "Blade Runner" was based on) Robotic animals, a wonderful part of the novel, was completely absent in the movie.

    Chris wrote: there is no emotional content whatsoever on the mirror world

    Star Trek gave us aliens that had no emotional content: Vulcans. There are Asimov's wonderful robot novels, and Bradbury dabbled with them too. In the context of making contact with aliens, I recall hearing some scientists mention the possibility that aliens may not have emotional content and thus might present a threat to us. Of course, emotions can also drive people to be amazingly destructive (that's the reason Vulcans evolved to get rid of emotions, according to Spock).

    Bottom line: There's no reason to think that emotions are required for consciousness.

    ReplyDelete
  174. A staggering 362 comments.

    If this is the new trend then you might need some AI to manage comment selection,)


    What would be the 3 laws of comment selection?


    (Inspired by the 3 laws of robotics):

    "
    A robot may not injure a human being or, through inaction, allow a human being to come to harm.

    A robot must obey orders given to it by human beings except where such orders would conflict with the First Law.

    A robot must protect its own existence as long as such protection does not conflict with the First or Second Law."

    ReplyDelete
  175. Dr. A.M. Castaldo wrote to me:
    >I put my view out there but don't expect many people to adopt it. Not because it is wrong, but because it doesn't cost them anything to believe whatever they want to believe, and to them believing that consciousness is magical, or mysterious, or special, or relies on a new mystery force that brings back paganism and imbues all things with "spirit" -- Well I think that's more fun for them. I'd rather know the truth, good or bad, but I know I'm in the minority.

    Well, I am as willing as the next physicist to be condescending towards philosophers! But, you know, they are really not all just people trying to bring back "paganism" as you seem to imply. Have you actually read David Chalmers' The Conscious Mind or, for an easier read, McGinn's The Mysterious Flame? These are serious, intelligent guys who are not neo-pagans or theosophists or theists. You might try understanding their point of view.

    In any case, I am still not clear as to how your proposed solution to the mind-brain problem gives a clear and definitive answer to the question of whether and under exactly what conditions machines can be conscious. Would you please fill me in on this or point me to the post in which you provided a clear explanation of all of this?

    Thanks.

    Dave

    ReplyDelete
  176. Steven Evans wrote to me (because I linked to a paper by Galen Strawson):
    >PhysicistDave, you need to change your user name to NotVeryGoodPhysicistDave.

    Steve, I indicated that I disagree with Strawson's conclusions but that I felt that he did point to one important distinction: "physical" in some unspecified, generic sense versus "physicSal" in the narrow sense of being explainable by current physics.

    It is an important distinction because too many discussions in this area throws around the word "physical" without ever making clear what exactly counts as "physical." Being clearer on this should help the discussion on both sides of the debate.

    And, I do kinda think you know way too little about my professional career to judge whether I am a "NotVeryGoodPhysicist." (Hint: I do have some patents involving applications of information theory to concrete problems; do you? I have published papers in HEP theory in major journals: have you? And, I have taken classes from three Nobel laureates: Feynman, Weinberg, and Thorne; have you?)

    Anyway, I suppose I could change my screen name to "NotSoGreatButKindaOkayPhysicistDave," but that is rather long. Incidentally, the screen name "PhysicistDave" does not make much sense in this forum where there are a number of physicists (it does make sense in some other forums where there are almost no other physicists), but the login procedure for Sabine's blog pushed me into using that persona.

    In any case, you reinforce my point that, for some reason, this topic really, really inflames negative, personally hostile feelings on both sides of the debate.

    All the best,

    Dave

    ReplyDelete
  177. Lawrence Crowell,

    1) Gödel's theorem is a result in mathematical logic that has no bearing on physics one way or the other. It is a limitation on the scope of the theorems of any consistent recursively axiomatizable theory of arithmetic. In general terms, it proves that there will always be arithmetic truths are that not theorems of such a system, which should surprise no one. It also shows that no sufficiently powerful consistent arithmetic theory can prove its own consistency, which is a little more interesting. But none of this tells us anything about physics, quantum or otherwise.

    2) The measurement problem is hardly insoluble: it has been solved several different ways, both with additional variables (Bohm) and with explicit and mathematically characterized collapses (GRW). None of these solutions has anything to do with consciousness or self-reference. It is just plain mathematical physics.

    3) The claim of Fauchiger and Renner has be shown to be incorrect by several different people independently of each other. There is a paper of Dustin Lazerovici and Mario Hubert (https://arxiv.org/abs/1809.08070) and another of Daniel Tausk (https://arxiv.org/abs/1812.11140). If you are interested in the argument and its shortcomings you can refer to either of these.

    ReplyDelete
  178. If the entire universe is composed of Artificial Intelligence, the idea of electrons thinking might not be as absurd as it first seems. These four scientists support the idea of the universe being composed of information/mathematics:

    a) The digital physics pioneered by Professor Edward Fredkin believes that biology reduces to chemistry reduces to physics reduces to the computation of information.(3)
    b) In 1990, John Wheeler (1911-2008) suggested that information is fundamental to the physics of the universe. According to this “it from bit” doctrine, all things physical are information-theoretic in origin. (4)
    c) Erik Verlinde says gravity is not a fundamental force of nature, but an emergent phenomenon. In the same way that temperature arises from the movement of microscopic particles, gravity emerges from the changes of fundamental bits of information, stored in the very structure of spacetime. (5)
    d) Cosmologist Max Tegmark hypothesizes that mathematical formulas create reality. (6)

    It seems plausible that the particular values of quantum spin could be determined by another set of particular values viz those in electronics’ binary digits, which always take the form of either 1 or 0. (Electronics could thus insert Artificial Intelligence and defiance of the Uncertainty Principle into everything from the subatomic scale through the biological to the astronomical.^) First, the 1’s and 0’s form the shape of a Mobius strip, which is merely two-dimensional (2-D).

    To use words from a recent science paper-
    In a holographic universe, all of the information in the universe is contained in 2D packages trillions of times smaller than an atom. (8)

    ^ Binary digits are proposed to be the Hidden Variables which “are an interpretation of quantum mechanics based on the belief that the theory is incomplete and that there is an underlying layer of reality that contains additional information about the quantum world. This extra information is in the form of the hidden variables, unseen but real quantities. The identification of these hidden variables would lead to exact predictions for the outcomes of measurements and not just probabilities of obtaining certain results.” (9)

    Then two strips must be joined to make a 4-D Klein bottle (10) which has length, width, depth and, when Wick rotation is programmed into the strips as a subroutine, the 4th dimension of movement in time. The type of Klein bottle formed would appear to be the figure-8 Klein. A diagram of many figure-8 Klein bottles would show that their positive curvature (on the spherical parts) fits together with their negative curvature (on saddle-shaped parts) to cancel and produce, on a cosmic scale, the flat curvature of space-time (11). When you have trillions of Mobius and figure-8 Klein elements assembled, you can follow the theory of the mass-giving Higgs field being the result of various couplings. (12) This theory has lost popularity since the Higgs boson was discovered. But an implication of a 1919 paper by Einstein called “Do gravitational fields play an essential role in the structure of elementary particles?” is that the coupling is between gravitons and photons. That could mean coupling is between the Mobius strip and the figure-8 Klein bottle (these exist on a level between photons/gravitons and 1’s/0’s, being built up into the particles and composed of the binary digits). With trillions of Mobius and figure-8 Klein elements assembled, these (now respectively called photons and gravitons) must interact via vector-tensor-scalar geometry to give matter what we call the emergent property of mass: similar to hydrogen and oxygen combining to give water what we call wetness. This proposed link between the Mobius strip and the Mobius doublet (figure-8 Klein bottle) would also be a link between the photon and graviton, suggesting unification of electromagnetism with gravitation. It also confirms Verlinde’s idea that gravity is an emergent property (emerging from maths).

    ReplyDelete
  179. Chris,

    "Consider Mary a bio-machine for detecting red. If I follow, Sabine is saying Mary's "experience" of detecting red is the machine's response for "I've detected red." In a very simple machine, a photocell with a red filter, a buzzer sounds for redness. In the Mary-machine, she "experiences" it.

    An experience isn't just a detection of some input. It's accompanied by the ability to put the input into context, connect it to memory, and to make "sense" of it. It's complicated. I'm not saying I know how it works. But, yes, of course what your brain does is all there is.

    "Others (full disclosure, including me) believe there may be a deeper mystery behind the subjective experience."

    I totally get that, but look, you either make that "mystery" one without evidence, so it's a type of religion. Or it's in conflict with data. Please be clear about it.

    ReplyDelete
  180. Quentin,

    First, your attitude towards panpsychism is all fine with me, except for the statement that:

    "it is a *metaphysical* explanation of why we have qualitative experience ("feels like")"

    I fail to see how postulating something explains it. Really what's the advantage over saying "God made it so"? That explains equally nothing.

    Second, I already said above that you can very well explain the relationship (as you emphasize) between brain patterns and experiences by the standard modes of the scientific method. I can repeat this once again if necessary.

    I have no doubt that what you say about Chalmers is correct. I don't find it useful, though, to mix up unsolvable with solvable problems.

    ReplyDelete
  181. Matthew,

    "Sabine.. You are not grasping the significance of Chalmers' "gap". Even if you map every neuronal activity in the entire brain you will never explain why the color red (to take but one example) looks as it does to the subject."

    You can map the neuronal activity and that *is* what the subject experiences when it looks at something. That is "why the color red looks as it does to the subject". It looks as it does because it gives rise to this particular brain response. And, as I said above, you can perfectly well explain this brain response if you have a suitable model (or call it a theory). Not that I say I have one. The point is simply that's a scientific question and can be answered by the usual scientific procedures.

    What you cannot do, and I have said this many times, is to reproduce the very same experience in your brain, because your brain is different. Hence, the best you can do is try to have an experience that is similar to that of the subject. You will never be able to find out.

    ReplyDelete
  182. Unknown,

    You missed the point. Look, you write:

    "Panpsychism offers a solution. Instead of assuming that consciousness is the product of processes within the brain, perhaps consciousness precedes processes within the brain, whereby the brain is simply a thinking, self-reflective, ego of a thing. "

    Now take your notion of unchanging "consciousness" that is the same for all particles (because you think it does not change) and try to explain anything with it. Go ahead. Tell us how come we are conscious based on your notion of conscious particles.

    ReplyDelete
  183. @Steven Mason: "Bottom line: There's no reason to think that emotions are required for consciousness."

    Because science fiction?

    ReplyDelete
  184. Bee, you write:

    "I'm afraid you still did not understand what I say. Unless you change something about the electron (or other particles) there are no "iotas" that can be "magnified through each level of combination". If you do not change the standard model, you have physics as we know and like it and that leaves explaining consciousness as hard as it has always been."

    We're missing each other on your OP critique and my response to it. I wrote to you previously that the flaw in your OP argument is your assumption that any iota of consciousness in an electron would lead to different types of particles being produced in the experiments you describe. Why would this be the necessary outcome of positing some kind of mentality in the electron? You don't explain this assumption very well, as opposed to other possible outcomes of rudimentary consciousness present in an electron, and I personally don't understand why you would make that assumption. "Change" means many different things, including position in space, which you don't seem to have considered. What Dyson is getting at with his quote about electrons and quantum theory more generally is that the individual outcomes and the consequent collective probabilistic distribution outcomes of QM are better explained as the product not of pure chance (another way of saying "we don't know") but of numerous highly rudimentary choices by each electron in each moment about where and how to manifest in the next moment.

    I don't think you responded to my comments on this point so let me ask you what you see as the logical difficulty in positing a highly rudimentary choicemaking ability in electrons and other particles with respect to two slit experiments, for example? I'm not asking whether you find this position plausible, just whether you see a logical problem here. This is no real explanation in Copenhagen or other interpretations of QM beyond the probabilistic randomness explanation. With respect to individual events, it seems just as logically viable to state (as Dyson, Bohm and others have) that there is a modicum of choice in those individual electrons in terms of how they choose to manifest in each moment, in response to the environment around them.

    ReplyDelete
  185. Sabine, you said
    ”An experience isn't just a detection of some input. It's accompanied by the ability to put the input into context, connect it to memory, and to make "sense" of it.”

    Does the “ability to put the input into context, connect it to memory” have to actually occur, or is the affordance of such ability sufficient? Can a detection of an input be an experience if there is no memory or “putting into context”?

    *
    [FWIW, I think the detection with the affordance is sufficient]
    [James of Seattle]

    ReplyDelete
  186. Hi Sabine.

    I've also posted the following comment on my blog: https://footnotes2plato.com/2019/01/06/electrons-dont-think-by-sabine-hossenfelder/

    I discovered your blog last night after Googling "Carlo Rovelli and Alfred North Whitehead." It brought me to Tam Hunt's interview with Rovelli. I have been studying Rovelli's popular works lately (I just finished The Order of Time) because I'd heard his loop quantum gravity might be a natural fit with Whitehead's panexperiential process-relational ontology. I am a philosopher, not a physicist or a mathematician, so I struggle with many technical papers in physics journals (it is helpful when the author is kind enough to lay out the conceptual structure of the maths). Luckily, i've noticed that popular books are the best place to look for a physicist's natural philosophy and the best way to understand the metaphysical background of a physicist's theories. I am looking forward to reading your book Lost in Math. It strikes me as another example of a larger trend in theoretical physics (also exemplified by Lee Smolin) that's challenging the ascendency of mathematical speculation over experimental and empirical grounding.

    As for your post "Electrons Don't Think", I don't know what panpsychist philosophy you read, but either it was badly written or you didn't understand it. There are, of course, many varieties of panpsychism, just like there are many varieties of materialism and idealism, etc. Perhaps the variety you read has misled you. The panpsychism of, for example, the mathematician, physicist, and philosopher Alfred North Whitehead was constructed precisely in order to provide a new metaphysical interpretation of the latest scientific evidence (including relativity, quantum, evolutionary, and complexity theories), since the old mechanistic materialism could no longer do the job in a coherent way. Panpsychism is metaphysics, not physics. A metaphysical scheme should aid in our philosophical interpretation of the physical evidence, not contradict it. Any philosopher whose metaphysics contradict the physical evidence is doing bad philosophy. 

    (part 2 follows in another post)

    ReplyDelete
  187. part 2:

    I like to distinguish between two main species of panpsychism:

    1) substance-property panpsychism (Aristotle, Spinoza, Leibniz, and contemporary philosophers Philip Goff, Galen Strawson, and David Chalmers seem to me to fall into this category) 

    2) process-relational panpsychism (Friedrich Schelling, William James, Henri Bergson, Gilles Deleuze, A. N. Whitehead)

    I count myself among the later category, and following the Whiteheadian philosopher David Ray Griffin, I prefer the term "panexperientialism" to panpsychism, since the idea is not that electrons have the full capacities of human psyches (reflective thinking, deliberate willing, artistic imagining, etc.) but that all self-organizing systems are possessed of at least some modicum of feeling, even if this feeling is faint and largely unconscious in the vast majority of systems. Human consciousness is an extremely rare and complex integration of the more primordial feelings of these self-organizing systems. 

    I unpack the differences between these species of panpsychism/panexperientialism at more length in this blog post. In short, the substance-property species of panpsychism has it that mind is an intrinsic property of all substance. This at least has the advantage over materialism that it avoids the hard problem of consciousness and provides a way out of the incoherence of dualism. But I think substance-property panpsychism is working with an overly abstract concept of consciousness. Consciousness is a relational process, not a quality inhering in a substance. Consciousness emerges between us, not in us.

    You write: panpsychism is "the idea that all matter – animate or inanimate – is conscious, we just happen to be somewhat more conscious than carrots. Panpsychism is the modern elan vital."

    I would say that panpsychism is the idea that all matter is animate. What is "matter," anyway, other than activity, energy vectors, vibrations? Is there really such a thing as "inanimate" matter, that is, stuff that just sits there and doesn't do anything? As for the "elan vital," I suppose you are trying to compare panpsychism to vitalism? Vitalism is the idea that some spiritual agency exists separately from a merely mechanistic material and drives it around; its the idea that, for example, angels are pushing the planets around in their orbits. The panexperientialist cosmology I articulate in my book Physics of the World-Soul explicitly denies this sort of dualism between spirit and matter. Panexperientialism is the idea that spirit and matter are not two, that mechanism is merely an appearance, a part mistaken for a self-existing whole, and that ultimately Nature is organic and animate from top to bottom. 

    ReplyDelete
  188. Tam,

    "what you see as the logical difficulty in positing a highly rudimentary choicemaking ability in electrons and other particles with respect to two slit experiments, for example? I'm not asking whether you find this position plausible, just whether you see a logical problem here."

    Define "choice". Define "choicemaking ability". Tell me how it explains anything better than particle physics without being in conflict with particle physics.

    "any iota of consciousness in an electron would lead to different types of particles being produced in the experiments you describe. Why would this be the necessary outcome of positing some kind of mentality in the electron?"

    Write it down, Tam. Write down a Lagrangian with "iotas of consciousness". Derive the standard model from it. Or if you do not like Lagrangians, try to write down any theory that does it.

    I am telling you that you either run into conflict with experiment, or end up with a theory that does nothing to explain consciousness. I already told you why, but I think you don't even understand the difficulty. It is not easy to change the standard model and still be compatible with the enormous number of experimental data.

    I'm not sure if you have read my book, but Arkani-Hamed goes on about exactly this. People don't seem to understand how hard it is to come up with any theory that fits all existing data. It's also why Brian Cox proclaimed that the LHC has rule out ghosts, which is another way of saying the same thing.

    ReplyDelete
  189. James,

    "Does the “ability to put the input into context, connect it to memory” have to actually occur, or is the affordance of such ability sufficient? Can a detection of an input be an experience if there is no memory or “putting into context”?"

    Really, I don't know. But I think that if a detection of input would count as "experience" then any microphone could be said to have "experiences" and that seems a rather useless terminology. You want something that connects with what humans commonly refer to as "experience", and that requires that you connect the input to your knowledge about your environment and yourself. I think you can have an "experience" without memory to put it into context (otherwise you couldn't experience new things), but if you have memory that will contribute to it (whether or not you like that).

    (That pure sensory input has limited relevance for experience can be demonstrated by very simple experiments. Eg, if you blindfold someone and tell them to bite into a sausage but give them a banana instead, they'll probably spit it out in disgust. Makes total sense, from an evolutionary perspective - also shows that memory and expectations are extremely important for experiences.)

    ReplyDelete
  190. Bee, I interviewed you about your book, remember, so yes I have read it :)

    To be clear on my point: I'm not suggesting any empirical difference in the Dysonian/Bohmian notion of electrons exerting choice in their positions in each moment. In this specific case, the empirical predictions are exactly the same. As I've mentioned before what I suggesting is a different interpretation of the formalisms, the way in which the maths map onto the real world. In Copenhagen, for example, the individual electron measurements on the measurement apparatus are the result of "chance" events. In this particular panpsychist interpretation of the maths, these events are due to choice. There is NO empirical difference. I want to be clear, however, that there may be different empirical outcomes in other aspects of physics, as Bohm discusses in his book Wholeness and the Implicate Order. But that isn't necessary or relevant to my particular point here.

    You will then say, again, well why posit a different interpretation if there's no particular empirical difference? My answer is, among other major benefits, the possibility of explaining human consciousness and consciousness more generally, by accepting that it's always associated with matter at least in rudimentary form, and it complexifies as matter complexifies. This is a very significant benefit of such an interpretation b/c as we've seen it is very difficult indeed to explain consciousness in the prevailing physics paradigm. As biologist Sewall Wright stated: "The emergence of mind from no-mind is sheer magic."

    The panpsychist view is the opposite of emergence; rather than emerging, consciousness is there all along in some form.

    ReplyDelete
  191. Tam,

    Right, now that you say it. Sorry. I get hundreds of comments a day, and sometimes I don't properly match names to people. Please don't take it personally.

    "the individual electron measurements on the measurement apparatus are the result of "chance" events. In this particular panpsychist interpretation of the maths, these events are due to choice. There is NO empirical difference... You will then say, again, well why posit a different interpretation if there's no particular empirical difference? My answer is, among other major benefits, the possibility of explaining human consciousness and consciousness more generally, by accepting that it's always associated with matter at least in rudimentary form, and it complexifies as matter complexifies."

    There, you did it once again. You start out with a theory that is identical to the standard model, and then a miracle happens and somehow it "explains" more than the standard model. Don't you see that that's a logical impossibility? More concretely, you state that "consciousness... complexifies as matter complexifies". Leaving aside that I am not sure what you mean by "complexifies" how do you think your panpsychisitic reinterpretation makes it any easier to explaining how this happens than would just starting with the standard model?

    ReplyDelete
  192. I think the higher the energy of the particle the easier it will be to see changes caused by thinking. An electron might take 1800 years to change itself whereas a proton might take a year because it 1800 times the mass -- thus energy and time speed. A higher mass human soul particle might change itself every 1/100 of a second. And for the universe as a whole as they say a day for us would be like a thousand years for the universe -- I think that is a very low estimate.

    ReplyDelete
  193. @PhysicistDave

    Strawson's paper is without value. The one piece of terminology you mention seems more like a petty dig to point out the "crazy" physicSalists don't know quite everything. It's strange that as a physicist you link to a crank paper like Strawson's. The argument in this blog post shows neatly and clearly that it is physicSally impossible for panpsychism to add anything to the explanation of consciousness from matter. But then links are posted to this neuroscientist or that philosopher making absolutely no sense, and posted by professional philosophers or academics published in Sci Am or physicists who have been taught by a triumvirate of Nobel laureates. It's the academic paper equivalent of Rickrolling - take a look at this paper - gotcha, it doesn't make any sense whatsoever. Strawsonlinking we can call it.

    Anyway, good luck with the physics. Keep away from the crank papers.

    ReplyDelete
  194. Sabine,

    This is not *my* attitude. As Philip Goff told you, this is how panpsychism must be understood in contemporary discussions. I'm not giving any personal opinion on the subject in my comments, only the state of the art.

    As for your remark on metaphysical explanations: well ok but 1/there are several different ways of saying "God made it so" (sic) and people in metaphysics are discussing which one makes more sense 2/you can dismiss the whole of metaphysics by the same kind of argument. No problem for me. Some people don't like metaphysics. But in history of science, some purely metaphysical discussions had an impact on the way to design new theories (ex. Substantial vs relational space, a purely metaphysical discussion that influenced the design of relativity theory) and in the long run it helped science. Perhaps contemporary discussions will have a similar impact for future ways of designing a theory of consciousness in neuroscience. Tononi's theory an example: it is based panpsychism. Certainly a substance dualist who believes human souls (a different way of saying "God made it so") would design different kind of theories, perhaps less successfully...

    That's all for me.

    Best,
    Quentin

    ReplyDelete
  195. Sabine wrote:
    >My best guess is that consciousness is a relational property that can emerge in system with many constituents that process information...

    Sabine, there are some very serious technical problems with taking "information" or "information processing" as somehow being the basis for consciousness, at least if the concept "information" is taken in the technical sense used in information theory.

    One big problem is that information theory hinges on the dichotomy between "information" and "noise": e.g. the fundamental result in information theory, Shannon's channel coding theorem, is only relevant if there is non-zero noise. But the distinction between information and noise is not a fundamental physical distinction but depends rather on the purpose of the analyst.

    A simple example of this is the analysis of certain "spread-spectrum" systems where you can take everyone else's "signals" to be your "noise." We all know of obvious examples of the same point in the history of physics: e.g., the CMB was initially considered unwanted and unexplained "noise" by Penzias and Wilson, whereas we now consider even tiny fluctuations in the CMB to contain important information.

    Of course, in some ultimate sense from the perspective of fundamental physics, we want everything to count as "information" and we want to consider nothing to be "noise" -- i.e., we want our theory to explain everything.

    But that includes too much as "information," at least if we want to use "information" to explain consciousness but still avoid panpsychism.

    The problem is that if we do not simply arbitrarily dismiss some information as noise, then a really big boulder is going to have more "information" in it than our brain has! After all, there are countless phonons banging around inside that boulder, and it would take many, many bits of information to completely characterize those phonons.

    Similarly, the boulder can be viewed as a computer that exquisitely carries out the extraordinarily difficult computational task of solving Schrödinger's equation for all the subatomic particles in the boulder.

    This is silly, of course, but it points to a real problem: how do we distinguish between the "real" information that is really being "processed' in our brain and that leads to consciousness vs. the "information processing" in the boulder that does not result in consciousness?

    I don't know if the panpsuychists have appealed to this argument, but they can give an easy answer: all information is equal and therefore the boulder is conscious!

    I realize that you are not actually arguing for the simple proposition that "consciousness = information processing." I think you are trying to say that somehow a full understanding of consciousness will connect it to information processing in some way we do not yet understand. That may well be true.

    But, I think it is true that an "information-processing" approach to consciousness that avoids panpsychism is going to have to be more complicated than many people realize.

    Again, for the record, I am very skeptical of panpsychism and somewhat doubtful about consciousness-as-information-processing. The one thing I am sure of is that I do not understand consciousness.

    Dave


    ReplyDelete
  196. A.M Castaldo,

    Sorry to respond bluntly but the discussion has been too long now and I'm tired of arguing...

    Thousands of very smart people around the world have written on the hard problem and are convinced it's a serious metaphysical problem. Many among them know neuroscience much better than we do here and have spent years pondering the issue.

    If you think it's "not that hard" or it's based on a simple confusion, I suggest you try to publish your thought in a philosophy journal. If it's well argumented it could certainly pass the review process. But take my advice: before doing that, read the thousands of people who have written on it... Just to be sure you understand the problem correctly.

    Best,
    Quentin

    ReplyDelete
  197. Note to all: that "hard" in "hard problem" should be understood as "not scientific, but metaphysical".

    ReplyDelete
  198. Physicist Dave,

    What I said doesn't depend on how you define "information." It's really about the ability of one system to act as a predictive model for another. You can quantify this without ever mentioning the word "information."

    ReplyDelete